Clin. Med. 3 Final

Réussis tes devoirs et examens dès maintenant avec Quizwiz!

50-year-old man is being seen in the office for follow-up of severe rheumatoid arthritis. He declines taking medications except for a daily multivitamin with iron. His laboratory examination results include a hemoglobin level of 9.8 g/dL. The colonoscopy this year was normal. Which of the following lab findings would most likely to be seen on laboratory workup of his anemia?

C.

A third-year medical student has been reading about the dangers of excessive anticoagulation and bleeding potential. He reviews the charts of several patients with AF currently taking warfarin. Which of the following patients is best suited to discontinue anticoagulation? a. A 45-year-old man who has normal echocardiographic findings and no history of heart disease or hypertension, but a family history of hyperlipidemia b. A 62-year-old man with chronic hypertension and dilated left atrium, but normal ejection fraction c. A 75-year-old woman who is in good health except for a prior stroke, from which she has recovered nearly all function d. A 52-year-old man with orthopnea and paroxysmal nocturnal dyspnea

a. A 45-year-old man who has normal echocardiographic findings and no history of heart disease or hypertension, but a family history of hyperlipidemia

Which one of the following patients should be promptly referred for endoscopy? a. A 65-year-old man with new onset of epigastric pain and weight loss b. A 32-year-old patient whose symptoms are not relieved with ranitidine c. A 29-year-old H. pylori-positive patient with dyspeptic symptoms d. A 49-year old woman with intermittent right upper quadrant pain following meals

a. A 65-year-old man with new onset of epigastric pain and weight loss

A 45-year-old woman with type 2 diabetes presents to the clinic for follow-up. She states that over the past year, she has decreased vision in the left eye. She also has had some occasional chest pain for the past 2 months. On examination, the blood pressure is 145/92 mm Hg. The cardiac and lung examinations are normal. Laboratory tests show the urinalysis reveals 1+ proteinuria. Serum laboratory tests reveal a baseline Cr of 1.6 mg/dL, a low-density lipoprotein (LDL) cholesterol level of 135 mg/dL. Which of the following is the best medication to start the patient on at this time? a. ACE inhibitor b. Hydralazine c. Oral nitrate d. Thiazide diuretic

a. ACE inhibitor

A 55-year-old man is noted to have moderately severe HF with impaired systolic function. Which of the following drugs would most likely lower his risk of mortality? a. ACE inhibitors b. Loop diuretics c. Digoxin d. Aspirin

a. ACE inhibitors

A 28-year-old woman presents to the emergency department with complaints of excessive bleeding from her gums and petechiae. She is otherwise healthy and takes no medications. She smokes half a pack of cigarettes daily and occasionally drinks beer. Her temperature is 99.1 °F, BP is 110/81 mm Hg, HR is 85 bpm, and respiratory rate (RR) is 12 breaths/min. On physical examination, she has bleeding from her gums and petechiae on her bilateral lower extremities. Her CBC shows a white blood cell (WBC) count of 87,000/mm3, a hemoglobin of 8.9 g/dL, and a platelet count of 22,000/mm3. Which of the following is the most likely etiology of her low platelet count? a. Acute leukemia b. Drug-induced thrombocytopenia c. Immune thrombocytopenia purpura d. Systemic lupus erythematosus

a. Acute leukemia

A 68-year-old man is found to have an incidental finding of anemia while hospitalized with pneumonia. His physical examination is normal except for crackles in the left lower lobe. Serum laboratory examinations reveal a normal MMA and a decreased serum folate level. Which of the following is the best next step? a. Administer CAGE questionnaire b. Esophagogastroduodenoscopy c. Serum iron assay d. Neurology consultation

a. Administer CAGE questionnaire

A 1-month-old boy has a fever to 102.7°F (39.3°C), is irritable, has diarrhea, and has not been eating well. On examination he has an immobile red TM that has pus behind it. Which of the following is the most appropriate course of action? a. Admission to the hospital with complete sepsis evaluation b. Intramuscular ceftriaxone and close outpatient follow-up c. Oral amoxicillin-clavulanate d. Oral cefuroxime e. High-dose oral amoxicillin

a. Admission to the hospital with complete sepsis evaluation

Which one of the following recommendations is accurate regarding the current USPSTF recommendation for osteoporosis screening in the elderly? a. All women with strong risk factors, regardless of age, should be screened for osteoporosis. b. Only women above the age of 65 years should be screened for osteoporosis. c. Men and women above the age of 65 years should be screened for osteoporosis. d. African American race is an independent risk factor for osteoporosis and should warrant screening regardless of other risk factors.

a. All women with strong risk factors, regardless of age, should be screened for osteoporosis.

Ototoxicity and nephrotoxicity are characteristic adverse effects of which of the following? a. Aminoglycosides b. B-Lactam antibiotics c. Chloramphenicol d. Fluoroquinolones

a. Aminoglycosides

A 49-year-old man is admitted to the intensive care unit with a diagnosis of an inferior myocardial infarction. His heart rate is 35 bpm, and blood pressure is 90/50 mm Hg. His ECG shows a Mobitz type I heart block. Which of the following is the best next step? a. Atropine b. Transvenous pacer c. Lidocaine d. Observation

a. Atropine

A 24-year-old man presents with increasing fatigue, easy bruising, and the diffuse formation of petechiae. His peripheral leukocyte count is increased, and multiple immature cells are present. A bone marrow biopsy is markedly hypercellular as a result of a diffuse proliferation of immature cells. The presence of which of the following abnormalities in these immature cells favors the diagnosis of AML rather than ALL? a. Auer rods b. Howell-Jolly bodies c. Pappenheimer bodies d. Reinke crystals e. Russell bodies

a. Auer rods

A 62-year-old asymptomatic woman is noted to have multiple myeloma and hypercalcemia but no bone lesions or end-organ damage. Which of the following therapies is useful for immediate treatment of the hypercalcemia? a. Bisphosphonates b. Erythropoietin c. Dexamethasone plus thalidomide d. Interferon-alpha e. Observe without treatment since she is asymptomatic

a. Bisphosphonates

Which of the following statements regarding available treatments for smoking cessation is accurate? a. Bupropion can be used in combination with nicotine supplements. b. Nicotine gum is most effective if chewed continuously, to promote a constant release of the nicotine. c. Nicotine supplements are most effective when used as needed for withdrawal symptoms. d. All of the available agents are more effective when used in combinations with each other.

a. Bupropion can be used in combination with nicotine supplements.

A pregnant woman who smokes one pack of cigarettes a day asks for your advice regarding smoking cessation while she is pregnant. Which of the following statements is most appropriate? a. Bupropion is pregnancy category C and relatively safe in pregnancy. b. Varenicline is pregnancy category B and relatively safe in pregnancy. c. Nicotine gum delivers a lower and safer dose of nicotine than the nasal spray. d. The use of smoking cessation products during pregnancy frequently leads to adverse outcomes.

a. Bupropion is pregnancy category C and relatively safe in pregnancy.

A father brings to the clinic his 6-year-old son who currently is undergoing induction chemotherapy for ALL. The school will not allow the child to register until his immunizations are up-to-date. Which of the following is the best course of action? a. Call the school nurse or principal to inform him or her that this child should not receive immunizations while he is taking chemotherapy. b. Update all immunizations except for measles, mumps, and rubella (MMR) and varicella. c. Update all immunizations except for oral polio vaccine. d. Update all immunizations. e. Call the school nurse or principal to inform him or her that this child will never receive immunizations because of the alteration in his immune system.

a. Call the school nurse or principal to inform him or her that this child should not receive immunizations while he is taking chemotherapy.

A 74-year-old man with no prior medical problems faints while shaving. He has a quick recovery and has no neurologic deficits. His blood sugar level is normal, and an ECG shows a normal sinus rhythm. Which of the following is the most useful diagnostic test of his probable condition? a. Carotid massage b. Echocardiogram c. Computed tomographic scan of the head d. Serial cardiac enzymes

a. Carotid massage

A 22-year-old woman presents with fatigue, arthralgias, and a nagging dry cough for the past 6 weeks, but no shortness of breath. On physical examination, her lungs are clear to auscultation, and she has bilateral pretibial tender erythematous raised nodules. Which of the following is your best next step? a. Chest radiograph b. High-resolution CT c. Empiric treatment for postnasal drip d. Antinuclear antibody test e. Initiation of antituberculosis therapy

a. Chest radiograph

A 77-year-old man is brought to your office by his wife, who states that he has been having mental difficulties in recent months, such as not being able to balance their checkbook or organize the paperwork for his annual visit with the accountant. She also tells you that he has reported seeing animals in the room with him and that he can describe them vividly. He takes frequent naps and stares blankly for long periods of time. He seems almost normal at times, but then he randomly appears very confused at other times. He has also been dreaming a lot and has fallen down more than once recently. He currently takes aspirin, 81 mg/d. On examination, the patient walks slowly with a stooped posture and almost falls when turning around. He has only minimal facial expressiveness. No tremor is noted, and the remainder of the examination is normal. He is able to recall three words out of three, but clock drawing is abnormal. Laboratory studies are normal, and CT of the brain shows changes of aging. What type of dementia does this patient most likely have? a. Dementia with Lewy bodies b. Alzheimer disease c. Frontotemporal dementia d. Vascular dementia e. Dementia of Parkinson disease

a. Dementia with Lewy bodies

A 36-year-old woman is brought into the emergency department for lethargy. She is unable to respond to commands but does open her eyes to painful stimuli. On examination, her HR is 110 bpm, BP is 90/60 mm Hg, and respiratory rate (RR) is 24 breaths/min. An arterial blood gas reveals a pH of 7.28, pco2 of 28 mm Hg, po2 of 90 mm Hg, and HCO3 of 22 mEq/L. Serum electrolytes reveal Na+ 138 mEq/L, K+ 3.8 mEq/L, Cl- 105 mEq/L, and HCO3 of 23 mEq/L. Which of the following is the most likely diagnosis that led to this patient's condition? a. Diarrhea b. Lactic acidosis c. Diabetic ketoacidosis d. Ethylene glycol ingestion

a. Diarrhea

A 55-year-old man is noted to have HF and states that he is comfortable at rest but becomes dyspneic when he walks to the bathroom. On echocardiography, he is noted to have an EF of 50%. Which of the following is the most accurate description of this patient's condition? a. Diastolic dysfunction (HF with preserved EF) b. Systolic dysfunction (HF with reduced EF) c. Dilated cardiomyopathy d. Pericardial disease

a. Diastolic dysfunction (HF with preserved EF)

A 28-year-old woman has been told she has rheumatic heart disease, specifically mitral stenosis. Which of the following murmurs is most likely present? a. Diastolic rumble at apex of the heart b. Early diastolic decrescendo at right upper sternal border c. Holosystolic murmur at apex d. Late-peaking systolic murmur at right upper sternal border

a. Diastolic rumble at apex of the heart

A 65-year-old man is being seen in the office for shortness of breath of 1 day's duration. He has smoked 1.5 packs of cigarettes per day for 35 years. On examination, he is cachectic and breathes with pursed lips. His chest is barrel-shaped, and the AP diameter is enlarged. Which of the following is the most likely physical exam finding in this patient? a. Diffuse expiratory wheezing b. Clubbing of the fingers c. Bibasilar inspiratory crackles with increased JVP d. Inspiratory stridor e. Third heart sound

a. Diffuse expiratory wheezing

A 25-year-old man with a history of a duodenal ulcer is being seen in the office for follow-up. He does not complain of abdominal pain and does not report any bloody stool or melena. His blood pressure (BP) is 120/80 mm Hg, heart rate (HR) is 80 beats per minute, and respiratory rate (RR) is 12 breaths/min. His hemoglobin level is 10 g/dL. Which of the following most likely will be seen on laboratory investigation? a. Elevated TIBC b. Mean corpuscular volume of 105 fL c. Normal serum ferritin d. Reticulocyte count of 4%

a. Elevated TIBC

A 42-year-old woman is noted to have Type II diabetes for 20 years. She is noted to have hypertension with BP in the 150/94 range. The urinalysis shows mild proteinuria. Which of the following drugs would be the best to treat the hypertension in this individual? a. Enalapril b. Propranolol c. Hydrochlorothiazide d. Nifedipine

a. Enalapril

A 22-year-old woman is pregnant at 14 weeks' gestation. She denies vaginal bleeding or prior medical problems. Her BP is 100/60 mm Hg, HR is 90 beats per minute, and RR is 14 breaths/min. Her hemoglobin level is 9 g/dL. She is counseled that the most likely cause of the low hemoglobin level is due to iron deficiency. She asks why she could have iron deficiency when she is no longer menstruating. Which of the following is the best explanation? a. Expanded blood volume and transport to the fetus b. Hemolysis c. Iron losses as a result of relative alkalosis of pregnancy d. Occult GI blood loss

a. Expanded blood volume and transport to the fetus

A 35-year-old man who is mildly obese has undertaken a strict fad diet for 3 months. He previously had been healthy but now complains of fatigue. His hemoglobin level is 10 g/dL, and his MCV is 105 fL. Which of the following is the most likely etiology of his anemia? a. Folate deficiency b. Iron deficiency c. Sideroblastic anemia d. Thalassemia e. Vitamin B12 deficiency

a. Folate deficiency

A 66-year-old man is taken to the emergency department due to new-onset seizures. He reports a severe headache over the past month and morning nausea and vomiting. He also has had some difficulty with walking and balance. On CT imaging, there is a 4-cm mass noted in the right parietal lobe of the brain. Which of the following is the most likely cell type of this brain mass? a. Glioblastoma multiforme (GBM) b. Lymphoma c. Schwannoma d. Medulloblastoma

a. Glioblastoma multiforme (GBM)

A 66-year-old woman is brought in to the office by her family because of difficulty with memory and being disoriented. These mental status changes have worsened over the past 6 months. A careful history and physical examination is performed, which confirms cognitive impairment but no focal neurological findings. Which of the following tests is most appropriate next diagnostic step in this patient? a. Head CT or MRI b. Lumbar puncture c. Rapid plasma reagin d. Electroencephalogram

a. Head CT or MRI

A 58-year-old Caucasian woman presents complaining of low back pain 4 weeks after a fall. She has no history of fever, unexplained weight loss, diabetes, or cancer. Her past medical history is significant for moderate persistent asthma and nicotine dependence. She had a hysterectomy for uterine fibroids at age 40. Which of the following aspects of the patient's history should prompt further evaluation of her pain? a. History of corticosteroid use b. Caucasian ethnicity c. Time course of back pain d. History of cocaine use e. Premenopausal age

a. History of corticosteroid use

A 64-year-old man sees you in the office because of a cough he has had for the past 4 days and a fever that started last night. He is short of breath and has significant malaise. He is a nonsmoker and has no history of lung disease. His medical history is significant for type 2 diabetes mellitus, which is well managed with medications and diet. A physical examination reveals an alert and mildly ill-appearing man who is speaking in complete sentences. His temperature is 38.1 °C (100.6 °F), pulse rate 95 beats/min, respiratory rate 22 breaths/min, blood pressure 115/70 mm Hg, and oxygen saturation 97% on room air. His heart has a regular rhythm, and respirations appear unlabored. He has rhonchi in the left lower lung field but has good air movement overall. A chest radiograph reveals a left lower lobe infiltrate. Which one of the following is the most appropriate setting for the management of this patient's pneumonia? a. Home with close monitoring b. An inpatient medical bed without telemetry monitoring c. An inpatient medical bed with telemetry monitoring d. An inpatient intensive care bed

a. Home with close monitoring

A 33-year-old man is diagnosed with essential hypertension. He is started on a blood pressure medication, and after 6 weeks, he notes fatigue, rash over his face, joint aches, and effusions. A serum antinuclear antibody (ANA) test is positive. Which of the following is the most likely agent? a. Hydralazine b. Propranolol c. Thiazide diuretic d. Nifedipine e. Enalapril

a. Hydralazine

A 32-year-old woman with a known history of sickle cell anemia presents to the emergency department for the fifth time this year, complaining of diffuse abdominal pain and severe bilateral lower extremity pain typical of her previous pain crises. Normally, she takes hydrocodone/acetaminophen every 8 hours, but for the past day she has required doses every 4 hours without relief. Which of the following therapies would most likely decrease her number of sickle cell crises? a. Hydroxyurea b. Folate supplementation c. Prophylactic penicillin d. Pneumococcal vaccination

a. Hydroxyurea

A 45-year-old man was brought to the emergency department after vomiting bright red blood. He has a blood pressure of 88/46 mm Hg and heart rate of 120 bpm. Which of the following is the best next step? a. Intravenous fluid resuscitation and preparation for a transfusion b. Administration of a PPI c. Guaiac test of the stool d. Treatment for H. pylori

a. Intravenous fluid resuscitation and preparation for a transfusion

A 58-year-old woman comes to your office complaining of fatigue. She has also noticed a burning sensation in her feet over the past 6 months. A CBC shows anemia with an increased MCV. Which of the following is the most likely cause of her anemia? a. Lack of intrinsic factor b. Inadequate dietary folate c. Strict vegetarian diet for the past 6 months d. Chronic GI blood lass

a. Lack of intrinsic factor

A 31-year-old woman with a long history of intermittent severe unilateral throbbing headache lasting hours to days associated with nausea and photophobia, but no preceding symptoms and no visual disturbance, occurring once or twice per month a. Migraine headache b. Tension headache c. Cluster headache d. Subarachnoid hemorrhage e. Meningitis

a. Migraine headache

A 68-year-old man has an incidental finding of anemia while in the hospital for alcohol abuse. a. Normal MMA; decreased serum folate level b. Elevated MMA; decreased serum B12 level c. Elevated ferritin; normal MCV; decreased serum iron level d. Decreased ferritin; decreased MCV; decreased serum iron level

a. Normal MMA; decreased serum folate level

A 56-year-old man with a medical history of hypertension, chronic kidney disease stage III, gout, and alcohol dependence is brought in by emergency medical services after being found down on the ground. He smells strongly of ethanol and has a prior history of delirium tremens based on previous medical records. He is admitted to the ICU with concern for alcohol withdrawal. His vitals on admission are: temperature 37 °C, BP 110/74 mm Hg, heart rate 77 bpm, RR 12 breaths/min, and SpO2 of 94% on room air. Due to concern for inability to protect his airway, he is intubated in the emergency room. The next day, he shows significant clinical improvement and is extubated. Later that day, he goes into withdrawal with altered mental status, and aspirates while eating. What is the next best step in management? a. Obtain chest x-rays and continue to monitor for symptoms. b. Obtain chest x-rays and start azithromycin therapy. c. Obtain chest x-rays and start vancomycin and cefepime. d. Obtain chest x-rays, perform bronchoscopy, and initiate steroid treatment.

a. Obtain chest x-rays and continue to monitor for symptoms.

A 32-year-old G2 P1 woman is pregnant at 12 weeks' gestation. She is being seen for her prenatal visit. As you examine her, you note some bruises on her arms and neck area. She looks down at the floor and is tearful when you ask about these bruises. She confides that her husband has been violent toward her. Which of the following factors is associated with an increased risk of intimate partner violence in this patient? a. Pregnancy b. Older age c. Higher income d. Married status

a. Pregnancy

A 25-year-old medical student is stuck with a hollow needle during a procedure performed on a patient known to have hepatitis B and C viral infections, but who is HIV negative. The student's baseline laboratory studies include serology: HBsAg negative, anti-HBsAb positive, and anti-HBc IgG negative. Which of the following regarding this medical student's hepatitis status is true? a. Prior vaccination with hepatitis B vaccine b. Acute infection with hepatitis B virus c. Prior infection with hepatitis B virus d. The student was vaccinated for hepatitis B but is not immune

a. Prior vaccination with hepatitis B vaccine

Which of the following represents the decrement in speech commonly exhibited by the patient with parkinsonism? a. Progressively inaudible speech b. Neologisms c. Expressive aphasia d. Receptive aphasia e. Word salad

a. Progressively inaudible speech

A 28-year-old woman is presenting to the primary care clinic for an annual physical. Her BMI is 48 kg/m2, blood pressure is 145/91 mm Hg, and Hb A1C is 8.4%. Her past medical history is remarkable for hypertension, dyslipidemia, DM, and obstructive sleep apnea. She is currently taking lisinopril, hydrochlorothiazide, atorvastatin, and insulin. The patient's weight has remained unchanged since the last visit despite starting moderate-intensity exercise and dietary modifications. The patient endorses chronic fatigue and daytime somnolence. Which of the following is the next best step for management of metabolic syndrome? a. Refer the patient for bariatric surgery b. Increase insulin dose c. Add nifedipine d. Prescribe continuous positive airway pressure (CPAP) therapy

a. Refer the patient for bariatric surgery

During the winter, a young day care worker develops watery diarrhea. Which of the following is the most likely etiology of the symptoms? a. Rotavirus b. Giardia c. Escherichia coli d. Staphylococcus aureus e. Cryptosporidium

a. Rotavirus

A 22-year-old woman presents with a worsening cough over 6 weeks that did not improve with a course of antibiotics or antitussives. Her serum calcium level is found to be 12.5 mg/dL, and a chest x-ray reveals bilateral hilar lymphadenopathy. She has erythema nodosum on her legs. Which of the following is the most likely diagnosis? a. Sarcoidosis b. Mycoplasma pneumonia c. Acute lymphoblastic leukemia d. Squamous cell carcinoma of the lung e. Pulmonary embolism

a. Sarcoidosis

A 6-year-old girl is brought to your office by her mother because of a rash first noticed a week ago. Her mother reports that several children in her child's school have chicken pox, but that her child has received all of her immunizations, including two doses of the varicella vaccine. You observe the child actively playing with the toys in the waiting room before both the mother and child are brought back to the examination room. The child has a temperature of 100.4 °F (38 °C), a pulse of 90 beats/min, a blood pressure of 100/70 mm Hg, and a respiratory rate of 20 breaths/min. The physical examination is unremarkable except for about 20 vesicles on erythematous bases sparsely scattered on the child's trunk and limbs. Which of the following is the most appropriate treatment? a. Supportive care b. Antiviral therapy c. Antibiotic therapy d. Immune globulin

a. Supportive care

A 13-year-old girl presents with fever and sore throat of 48 hours' duration. She has a temperature of 101 °F in the office and is tachycardic, with a pulse of 118 beats/min. Her physical examination is positive for tender, enlarged left cervical lymphadenopathy and tachycardia. Her pharynx is erythematous with tonsillar enlargement or exudate. She has had no cough. What is the best step in management? a. Treat empirically with antibiotics b. Order rapid strep test and, if positive, treat with antibiotics c. Neither further testing nor antibiotics d. Order throat culture and, if positive, treat with antibiotics

a. Treat empirically with antibiotics

A 25-year-old woman who is training for a competitive marathon complains of "hitting a wall" and "getting short of breath quicker than [she] should." She complains of coughing at the end of her training runs and states that she may be expecting too much of herself. She does not smoke and has no significant family history or history of occupational or environmental exposures. Her physical findings, including lung examination, are unremarkable. Spirometry reveals normal values both pre- and postalbuterol treatment. What would be the most reasonable first step in the treatment of this patient? a. Trial of albuterol MDI before exercise b. Chest radiograph c. Chest CT d. Counseling for athletic burnout or stress e. An echocardiogram to rule out pulmonary hypertension or cardiac disorder

a. Trial of albuterol MDI before exercise

A 67-year-old woman is being seen in the office for the acute onset of dizziness. She says she was sitting on the couch watching TV when the episode came on, and the room was spinning. She feels nauseous. Her history includes a stroke 2 years ago that left some right-sided weakness. Which of the following arteries is most likely to be affected in today's presentation? a. Vertebrobasilar b. Carotid c. Aorta d. Middle cerebral

a. Vertebrobasilar

A 42-year-old woman with a past medical history of gestational diabetes during her last pregnancy 6 years previously usually has screening for type 2 diabetes every year. Which of the following screening methods has the highest sensitivity for diagnosing type 2 diabetes? a. Fasting serum glucose b. 2-hour glucose intolerance test c. Hemoglobin A1c d. Random glucose

b. 2-hour glucose intolerance test

Which of the following asymptomatic patients would most benefit from treatment of the finding of more than 105 CFU/mL of Escherichia coli on urine culture? a. A 23-year-old sexually active woman b. A 33-year-old pregnant woman c. A 53-year-old diabetic woman d. A 73-year-old woman in a nursing home

b. A 33-year-old pregnant woman

A 37-year-old woman was being followed by her primary provider for symptomatic gallstones, which were confirmed on ultrasonography. She was placed on a low-fat diet and was doing well for the past 3 months. However, today, she presents to the emergency center with severe RUQ pain and nausea. On examination, her temperature is 102.3 °F, heart rate 100 bpm, and blood pressure 120/70 mm Hg. Her abdominal examination reveals marked RUQ tenderness and guarding. There is no rebound tenderness. Which of the following is the most likely diagnosis? a. Acute cholangitis b. Acute cholecystitis c. Acute pancreatitis d. Acute perforation of the gallbladder

b. Acute cholecystitis

An 85-year-old nursing home resident with a medical history of diastolic heart failure, hypertension, diabetes mellitus, and dementia requiring assistance in all activities of daily life presents with a 3-day history of fever and nonbloody, productive cough. Her mental status was found to be more altered than usual. On admission, her vital signs show the following: temperature of 39 °C, BP 105/70 mm Hg, heart rate 93 bpm, RR 32 breaths/min, and SpO2 94% on room air. Chest x-ray reveals a right middle lobe consolidation. Which of the following is the best medical treatment of the patient? a. Admit to the floor and start intravenous cefepime and linezolid. b. Admit to the floor and start intravenous azithromycin and ceftriaxone. c. Admit to the ICU and start intravenous vancomycin and linezolid. d. Discharge home with a prescription of oral amoxicillin and cefpodoxime. e. Discharge home with oral azithromycin and oral cefpodoxime.

b. Admit to the floor and start intravenous azithromycin and ceftriaxone.

A 21-year-old previously healthy woman has been hospitalized for 3 weeks for psychiatric reasons. She developed auditory hallucinations and persecutory delusions over the course of 3 days. Since hospitalization, she was started on haloperidol 2 mg three times daily. Within a week of treatment, she developed stooped posture and a shuffling gait. Her head was slightly tremulous, and her movements became slowed. Her medication was changed to thioridazine, and trihexyphenidyl was added. Over the next 2 weeks, she became much more animated and reported no recurrence of her hallucinations. Which of the following is the most likely diagnosis responsible for the gait and posture condition? a. Hyperparathyroidism b. Adverse effect of neuroleptic c. Encephalitis d. Hypermagnesemia e. Tourette syndrome

b. Adverse effect of neuroleptic

In the United States, which of the following is most likely to have caused the HF in the patient described in Question 4.1? a. Diabetes b. Atherosclerosis c. Alcohol d. Rheumatic heart disease

b. Atherosclerosis

A 62-year-old man is being seen in the office for establishment of care. He reports that he had an acute myocardial infarction (MI) 1 year previously and lost his insurance and has not been able to take any medications since his heart attack. He smokes half a pack a day, which is less than the 1 pack per day previously. On examination, his blood pressure is 110/60 mm Hg. The cardiac examination is normal, and there is no jugular vein distension or pedal edema. His LDL-C is 105 mg/dL, HDL cholesterol is 28 mg/dL, and total cholesterol is 170 mg/dL. According to the NICE guidelines, what medication therapy should be initiated at this time? a. Ezetimibe b. Atorvastatin c. Niacin d. Gemfibrozil

b. Atorvastatin

A 45-year-old woman complains of recurring episodes of vertigo. Her spells last 10 to 15 seconds and occur mostly when rolling out of bed (and once when changing a light bulb). She denies hearing loss or tinnitus. Her physical examination is normal except for rotatory nystagmus when she is lying with the right ear down. What is the most likely diagnosis? a. Acute serous labyrinthitis b. BPPV c. Vestibular migraine d. Ménière disease e. Viral vestibular neuronitis

b. BPPV

A 60-year-old man is diagnosed with moderately severe COPD. He admits to a long history of cigarette smoking and is still currently smoking one pack per day. In counseling him about the benefits of smoking cessation, which of the following statements is most accurate? a. By quitting, his pulmonary function will significantly improve. b. By quitting, his current pulmonary function will be unchanged, but the rate of pulmonary function decline will slow. c. By quitting, his current pulmonary function and the rate of decline are unchanged, but there are cardiovascular benefits. d. By quitting, his age-related decline in pulmonary function will slow but will not approach that of a nonsmoker of the same age.

b. By quitting, his current pulmonary function will be unchanged, but the rate of pulmonary function decline will slow.

A 24-year-old intravenous drug user is admitted to the hospital with 4 weeks of fever. He has three blood cultures positive for growth of Candida species. After 2 days in the hospital, he develops a cold, blue right great toe. Which of the following is the appropriate next step? a. Repeat echocardiography to see if the large aortic vegetation previously seen has now embolized. b. Cardiovascular surgery consultation for aortic valve replacement. c. Aortic angiography to evaluate for a mycotic aneurysm, which may be embolizing. d. Switch from fluconazole to amphotericin B.

b. Cardiovascular surgery consultation for aortic valve replacement.

A previously healthy 42-year-old overweight woman presents to the emergency department complaining of sudden, right upper abdominal pain that "comes and goes," lasting approximately 45 minutes after eating at a fast-food restaurant. The patient has vomited twice since the pain started, and any attempt to eat worsens the pain. She is sexually active with one male partner, uses condoms regularly, and denies drug use. Which of the following is the most likely cause? a. Gastric ulcer b. Cholelithiasis c. Duodenal ulcer d. Acute hepatitis

b. Cholelithiasis

A 65-year-old man with a history of rheumatoid arthritis is found to have a microcytic anemia. He had a colonoscopy 1 year ago that was normal, and a stool guaiac is negative. Which of the following is the most likely cause of his anemia? a. Iron deficiency b. Chronic disease c. Pernicious anemia d. Folate deficiency

b. Chronic disease

A mother brings to the clinic her 4-year-old son who began complaining of right knee pain 2 weeks ago, is limping slightly, is fatigued, and has had a fever to 100.4°F (38°C). Which of the following laboratory tests is most important? a. Antinuclear antibodies b. Complete blood count (CBC) with differential and platelets c. Epstein-Barr virus titer d. Rheumatoid factor e. Sedimentation rate

b. Complete blood count (CBC) with differential and platelets

A 68-year-old man is brought into the clinic due to the patient having difficulty with his memory and with cooking for and shopping for himself. His wife passed away 2 years previously. He often becomes confused about where he is and gets lost when he takes a walk in the neighborhood. He is diagnosed with AD. A CT scan of the brain is performed. Which of the following is most likely to be seen on imaging? a. Chronic subdural hematoma b. Cortical atrophy with atrophy of medial temporal structures c. Ventriculomegaly without cortical atrophy d. Normal cerebral ventricles and normal brain tissue, acetylcholine deficiency

b. Cortical atrophy with atrophy of medial temporal structures

A 48-year-old woman is noted to have AF with a ventricular rate of 140 bpm. She is feeling dizzy and dyspneic, with a systolic blood pressure of 75/48 mm Hg. Which of the following is the most appropriate next step? a. Intravenous digoxin b. DC cardioversion c. Vagal maneuvers d. Intravenous diltiazem

b. DC cardioversion

A 67-year-old woman with known Parkinson disease is brought to the clinic by her health care provider. She is confined to a wheelchair and completely dependent on others. You notice large, grossly abnormal movements in both arms and legs. The patient has to be strapped to the wheelchair to avoid falling out and cannot keep her shoes on due to the jerking movements. Bed rails have had to be installed on her bed to prevent her from falling out at night. She is not able to tell the correct month or the year. She has not had a change in her medication in 6 months. Which of the following medication adjustments would benefit her most? a. Add haloperidol b. Decrease levodopa/carbidopa c. Increase levodopa/carbidopa d. Add donepezil e. Add entacapone

b. Decrease levodopa/carbidopa

A 68-year-old man with a history of end-stage renal disease is admitted to the hospital for chest pain. On examination, a pericardial friction rub is noted. His ECG shows diffuse ST-segment elevation. Which of the following is the best definitive treatment? a. Nonsteroidal anti-inflammatory drugs b. Dialysis c. Steroids d. Sodium polystyrene sulfonate (Kayexalate)

b. Dialysis

A 67-year-old man with coronary artery disease, dyslipidemia, and eczema comes to you complaining of lower back pain and left leg pain. The pain is worse when he stands for long periods of time but improves when he bends forward to push his shopping cart around the grocery store. He indicates that his feet "burn" and "ache" after walking different distances every day. His lower extremity neuromuscular examination is unremarkable. Which of the following is the most appropriate treatment for this patient? a. Emergent spinal cord decompression b. Epidural corticosteroid injection c. Kyphoplasty d. Bed rest for 4 days e. Tramadol

b. Epidural corticosteroid injection

A 56-year-old man is admitted to the hospital for chest pain of 2-hour duration. His heart rate is 42 bpm, with sinus bradycardia on ECG, as well as ST-segment elevation in leads II, III, and aVF. Which of the following is the most likely diagnosis? a. Left-circumflex territory infarction b. Inferior wall infarction c. LV aneurysm d. Anterior wall infarction

b. Inferior wall infarction

A 36-year-old woman has severe burning chest pain that radiates to her neck. The pain occurs particularly after meals, especially when she lies down, and is not precipitated by exertion. She is admitted for observation. Serial ECG and troponin I levels are normal. Which of the following is the best next step? a. Stress thallium treadmill test b. Initiation of a proton pump inhibitor c. Coronary angiography d. Initiation of an antidepressant such as a selective serotonin reuptake inhibitor (SSRI)

b. Initiation of a proton pump inhibitor

An 11-year-old girl is being seen in the pediatric emergency department for near syncope. She is being followed for type 1 diabetes and is not very adherent to her medications or diet. Her current BP is 90/60 mm Hg, HR is 120 bpm, RR is 26 breaths/min, and temperature is afebrile. Based on an arterial blood gas with a pH of 7.23 and a serum glucose level of 550 mg/dL, she is diagnosed with DKA. Which of the following is the most important first step in the treatment of this patient? a. Replacement of potassium b. Intravenous fluid replacement c. Replacement of phosphorus d. Antibiotic therapy

b. Intravenous fluid replacement

A 74-year-old man was noted to have excellent cognitive and motor skills 12 months ago. His wife noted that 6 months ago his function deteriorated noticeably of somewhat sudden onset, and he seemed to be at this new baseline until he became worse 2 months ago. Which of the following is most likely to reveal the etiology of his functional decline? a. HIV antibody test b. Magnetic resonance imaging of the brain c. Cerebrospinal fluid (CSF) Venereal Disease Research Laboratory (VDRL) test d. Serum thyroid-stimulating hormone

b. Magnetic resonance imaging of the brain

A 69-year-old man is being seen in the emergency center for gradually worsening dyspnea and a nagging cough over the past 3 months. He denied feeling warm or having fever. The chest radiograph shows a right-sided pleural effusion. A diagnostic thoracentesis reveals gross blood in the fluid. Which of the following is the most likely diagnosis? a. Parapneumonic effusion b. Malignancy in the pleural space c. Rupture of aortic dissection into the pleural space d. Pulmonary embolism with pulmonary infarction

b. Malignancy in the pleural space

Three days after beginning oral amoxicillin therapy for otitis media (OM), a 4-year-old boy has continued fever, ear pain, and swelling with redness behind his ear. His ear lobe is pushed superiorly and laterally. He seems to be doing well otherwise. Which of the following is the most appropriate course of action? a. Change to oral amoxicillin-clavulanate b. Myringotomy and parenteral antibiotics c. Nuclear scan of the head d. Topical steroids e. Tympanocentesis

b. Myringotomy and parenteral antibiotics

A 63-year-old man is brought in by his family because of memory loss. They have noted a worsening of his symptoms over several months. They also report that he has had multiple falls, hitting his head on one occasion, and has had frequent urinary incontinence. On examination, a gait apraxia (slow shuffling steps) is noted. Which of the following is the most likely diagnosis? a. Alzheimer disease b. Normal pressure hydrocephalus c. Dementia with Lewy bodies d. Delirium

b. Normal pressure hydrocephalus

In a suicide attempt, an 18-year-old woman took 4 g of acetaminophen, approximately 8 hours previously. On examination, she is drowsy but oriented and answers questions appropriately. Her blood pressure is 120/70 mm Hg and heart rate is 90 bpm. She is anicteric. Her heart, lung, and abdominal examinations are normal. Her acetaminophen level is 30 mcg/mL. Which of the following is the best next step to be performed for this patient? a. Immediately start NAC b. Observation c. Alkalinize the urine d. Administer intravenous activated charcoal

b. Observation

A 56-year-old man with known CKD presents to the emergency center with a 3-day history of shortness of breath and rapid weight gain. On examination of the heart, you are able to auscultate an S3 heart sound and hear crackles at the lung bases. You also see moderate JVD. The oxygen saturation is 90% on room air. Which of the following is your next step in evaluation? a. Order an echocardiogram. b. Order a chest x-ray. c. Measure a Cr to calculate GFR. d. Order a computed tomographic (CT) angiograph of the chest.

b. Order a chest x-ray.

On routine blood work performed for a life insurance application, a 48-year-old premenopausal woman was found to have a calcium level of 12 mg/dL (normal 8.8-10.4 mg/dL) and a phosphate level of 2 mg/dL (normal 3.0-4.5 mg/dL). She is not anemic and has no symptoms. Her medical history is significant for osteoporosis, which was discovered on a dual-energy x-ray absorptiometry (DEXA) scan performed last year. Which of the following is the most likely cause of her hypercalcemia? a. Multiple myeloma b. Parathyroid adenoma c. Familial hypocalciuric hypercalcemia d. Sarcoidosis e. Undiagnosed breast cancer

b. Parathyroid adenoma

A 4-year-old boy is brought to your office by his mother for evaluation of a rash on his face that his mother first noticed one day previously. His mother comments that it looks like somebody "slapped him." The mother reports that he has had a cold for the last couple of days. The child's physical examination is unremarkable except for an erythematous macular rash over both cheeks and a temperature of 102 °F (38.9 °C). The mother admits that the child is behind on his immunization schedule. Which of the following is the most likely cause? a. Varicella zoster virus b. Parvovirus B19 c. Human herpesvirus 6 d. Rubella virus e. Child abuse warranting immediate contact with social services

b. Parvovirus B19

A 55-year-old man with HF presents to the emergency department with a 1-week history of dyspnea on exertion and swelling in both ankles. He has had no fever or cough. Chest radiography shows bilateral pleural effusions. Which of the following is the most likely pleural fluid characteristic if thoracentesis is performed? a. Pleural fluid LDH 39, LDH ratio 0.2, protein ratio 0.7 b. Pleural fluid LDH 39, LDH ratio 0.2, protein ratio 0.1 c. Pleural fluid LDH 599, LDH ratio 0.9, protein ratio 0.1 d. Pleural fluid LDH 599, LDH ratio 0.9, protein ratio 0.7

b. Pleural fluid LDH 39, LDH ration 0.2, protein ratio 0.1

A 5-year-old boy is brought to the clinic with cough, congestion, runny nose, and low-grade fever for the past 3 days. He has been treated with acetaminophen for fever and given honey for his cough. He has been eating well and has been active at home. On examination, his temperature is 99.7 °F, and his vital signs are otherwise normal. He is active, playful, and smiles during the encounter. You note that his right tympanic membrane is red and bulging, but the left tympanic membrane is normal. His examination is otherwise normal. At this time, the best step in management is which of the following? a. Prescription for amoxicillin for OM b. Reassurance with a follow-up visit if not improving in the next 10 days c. Referral for a hearing test d. Prescription for ciprofloxacin otic drops for OE

b. Reassurance with a follow-up visit if not improving in the next 10 days

A 45-year-old man comes in for counseling about his fasting plasma glucose test after a routine health maintenance examination. On two separate occasions, the result has been 115 mg/dL and 120 mg/dL. Which of the following is the most appropriate next step at this time? a. Reassurance that these are normal blood sugars. b. Recommend weight loss, an ADA diet, and exercise. c. Diagnose diabetes mellitus and start on a sulfonylurea. d. Recommend cardiac stress testing. e. Obtain stat arterial blood gas and serum ketone levels.

b. Recommend weight loss, an ADA diet, and exercise.

A 63-year-old woman with a history of cervical cancer treated with a hysterectomy and pelvic irradiation now presents with acute oliguric renal failure. On physical examination, she has normal jugular venous pressure, is normotensive without orthostasis, and has a benign abdominal examination. Her urinalysis shows a specific gravity of 1.010, with no cells or casts on microscopy. Urinary FENa is 2%, and the Na level is 35 mEq/L. Which of the following is the best next step? a. Bolus of intravenous fluids b. Renal ultrasound c. Computed tomographic scan of the abdomen with intravenous contrast d. Administration of furosemide to increase her urine output

b. Renal ultrasound

A 62-year-old woman presents for a routine physical examination. She is asymptomatic and is not taking any medications. Her BP is found to be 145/85 mm Hg on two readings, and her BMI is 29 kg/m2. Review of her chart reveals that her BP was 143/84 mm Hg on a visit 4 months ago for a urinary tract infection. Which of the following is the most accurate statement regarding her BP? a. Her BP is normal, and she is at average risk for developing hypertension. b. She is at risk for needing pharmacologic treatment for hypertension. c. She has hypertension and should be started on thiazide diuretic. d. She has hypertension and should be started on multidrug therapy.

b. She is at risk for needing pharmacologic treatment for hypertension.

A 55-year-old woman presents to your office for follow-up. She was discharged from the hospital 1 week ago following an acute MI. She has quit smoking since that time and vows to stay off cigarettes forever. Her lipid levels are total cholesterol 240 mg/dL, HDL 50 mg/dL, LDL-C 160 mg/dL, and triglycerides 150 mg/dL. In addition to initiation of TLCs, which of the following is the most appropriate management at this time? a. Nothing further b. Start on a statin c. Start on nicotinic acid d. Start on a statin and nicotinic acid

b. Start on a statin

A 20-year-old woman is found to be anemic (10 g/dL) on routine laboratory tests. She is otherwise healthy, with review of systems notable only for heavy menses. Iron studies are as follows: decreased MCV, decreased ferritin, increased TIBC, and increased RDW. Which of the following is the best next step? a. Transfuse one unit of packed RBCs b. Start oral ferrous sulfate 325mg twice daily c. Refer for colonoscopy d. Start oral folic acid 1mg daily

b. Start oral ferrous sulfate 325mg twice daily

A 62-year-old man with recently diagnosed chronic obstructive pulmonary disease presents to your office in November for a routine examination. He has not had any immunizations in more than 10 years. Which of the following immunizations would be most appropriate for this individual? a. Tetanus-diphtheria only b. Tdap, pneumococcal, and influenza c. Pneumococcal and influenza d. Tdap, pneumococcal, influenza, and meningococcal

b. Tdap, pneumococcal, and influenza

A 5-year-old child is brought into the office due to the mother's concern about difficulty breathing. On examination, the child appears toxic and has a high fever, cough productive of thick mucopurulent expectorant, and stridor with wheezing. The child is up to date regarding immunizations. Which of the following are the most likely condition and causative organism requiring antibiotic therapy? a. Epiglottitis due to H. influenzae b. Tracheitis due to S. aureus c. Epiglottitis due to S. pyogenes (beta-hemolytic streptococcus group A) d. Tracheitis due to S. pyogenes (beta-hemolytic streptococcus group A) e. Retropharyngeal abscess due to S. aureus

b. Tracheitis due to S. aureus

A 62-year-old diabetic man underwent an abdominal aortic aneurysm repair 2 days ago. He is being treated with gentamicin for a urinary tract infection. His urine output has fallen to 300 mL over 24 hours, and his serum creatinine has risen from 1.1 mg/dL on admission to 1.9 mg/dL. Which of the following laboratory values would be most consistent with a prerenal etiology of his renal insufficiency? a. FENa of 3% b. Urinary sodium level of 10mEq/L c. Central venous pressure reading of 10 mmHg d. Gentamicin trough level of 4mg/mL

b. Urinary sodium level of 10mEq/L

A 38-year-old woman presents with increasing weakness and lethargy. Her peripheral white cell count is markedly elevated, and her leukocyte alkaline phosphatase score is markedly decreased. Which of the following chromosomal translocations is most consistent with a diagnosis of chronic myelocytic leukemia? a. t(8;14) b. t(9;22) c. t(11;14) d. t(14;18) e. t(15;17)

b. t(9;22)

A 48-year-old woman presents for an annual examination. She has had a supracervical hysterectomy and Pap smears with HPV cotesting every 5 years since her 20s and all were normal. She read on the Internet that women who have had a hysterectomy no longer require Pap smears. Which of the following would be your advice? a. "You no longer need to get Pap smears since you have had a hysterectomy." b. "You should continue to have Pap smears every 3 years since your hysterectomy is an indication to shorten the interval for testing." c. "You should continue to have Pap smears with HPV cotesting every 5 years since your hysterectomy does not exclude you from routine screening recommendation in your age group." d. "You should continue with annual Pap smears until the age of 50. If they are all normal, you can stop having them at that time."

c. "You should continue to have Pap smears with HPV cotesting every 5 years since your hysterectomy does not exclude you from routine screening recommendation in your age group."

A 48-year-old Caucasian man with type 2 diabetes has had persistent BP readings of 150/95 mm Hg for the past 6 months. Current medications include glyburide and metformin. His last hemoglobin A1c was 7.9%, and the patient has a BMI of 24 kg/m2. On physical examination, position sense is intact but a peripheral neuropathy is detected in a stocking-and-glove pattern. Vibratory sensation is decreased bilaterally on both lower extremities. Eye examination shows mild papilledema but no cotton wool spots. When questioned, he says that he still occasionally sneaks a cookie after dinner and drinks alcohol nightly. Which of the following is the most appropriate treatment for him? a. DASH diet and recheck BP in 3 months b. Thiazide diuretic alone c. ACE inhibitor alone d. Combination of ACE inhibitor and thiazide diuretic

c. ACE inhibitor alone

A 45-year-old obese woman presents for follow-up for her diabetes. She currently takes metformin 1000 mg twice per day, and her fasting morning glucose runs approximately 170 to 200 mg/dL. Her last serum Hb A1C level was 7.9%. She states that she conscientiously follows her diet and walks 30 minutes to 1 hour daily. Which of the following is the best next step in her care? a. Refer to an endocrinologist for an insulin pump. b. Stop metformin and start on glimepiride. c. Add a second agent for the treatment of her diabetes. d. Hospitalize her urgently.

c. Add a second agent for the treatment of her diabetes.

A 38-year-old woman presents with progressively worsening dyspnea and cough. She has never smoked cigarettes, has no known passive smoke exposure, and does not have any occupational exposure to chemicals. She has a family history of cirrhosis. Pulmonary function testing shows obstructive lung disease that does not respond to bronchodilators, and a complete metabolic panel demonstrates elevation of alanine aminotransferase (ALT) and aspartate aminotransferase (AST). Which of the following is the most likely etiology? a. Radon exposure at home b. Neoplasm c. Alpha-1 antitrypsin deficiency d. Pulmonary hypertension

c. Alpha-1 antitrypsin deficiency

A 36-year-old man is noted to have a bothersome "mole" that on biopsy reveals malignant melanoma. The pathologist comments that this histology is a very rare type of melanoma and usually escapes diagnosis until a more advanced stage. Which of the following is the most likely diagnosis? a. Melanoma in situ b. Superficial spreading melanoma c. Amelanotic melanoma d. Nodular melanoma

c. Amelanotic melanoma

A 12-year-old adolescent male presents with eye itching and redness. He has clear drainage from his eyes but no crusting. Examination today is normal except for mildly injected conjunctiva bilaterally. Which of the following is the most appropriate treatment? a. Antibiotic eye drops b. Ophthalmology consultation c. Anti-inflammatory eye drops d. Oral leukotriene inhibitor

c. Anti-inflammatory eye drops

A 59-year-old male smoker complains of severe retrosternal squeezing chest pain of 30 minutes' duration. The paramedics have given sublingual nitroglycerin and oxygen by nasal cannula. His blood pressure is 110/70 mm Hg, and heart rate is 90 bpm on arrival to the Emergency Department. The ECG is normal. Which of the following is the best next step? a. Echocardiography b. Thallium stress test c. Aspirin d. Coronary angiography e. Coronary artery bypass

c. Aspirin

A 67-year-old woman has diabetes and mild hypertension. She is noted to have diabetic retinopathy, and she states that she cannot feel her legs. She has recurrent episodes of light-headedness when she gets up in the morning. She comes in now because she fainted this morning. Which of the following is the most likely cause of her syncope? a. Carotid sinus hypersensitivity b. Pulmonary embolism c. Autonomic neuropathy d. Critical aortic stenosis

c. Autonomic neuropathy

A 57-year-old man is brought into the emergency center for shortness of breath and light-headedness. He is found to have a blood pressure of 68/50 mm Hg and heart rate of 140 bpm. His jugular venous pulses are elevated. The lungs have inspiratory crackles on examination. All four extremities are cold and clammy. Which of the following is the most likely etiology for this patient's condition? a. Septic shock b. Adrenal crisis c. Cardiogenic shock d. Hypovolemic shock

c. Cardiogenic shock

A previously healthy 75-year-old man is being seen in the office for an episode of weakness that he reports occurred 2 weeks previously. He states that he had weakness of the left arm and left leg that lasted for 4 hours and slowly resolved over the rest of day. His arm and leg also felt "heavy" and had some numbness. He denies having similar episodes in the past. On physical examination, his BP is 140/90 mm Hg. He is found to have a right carotid bruit on auscultation. A duplex ultrasound demonstrates a 75% stenosis of the right carotid artery. Which of the following is the best therapy for this patient at this time? a. Aspirin plus clopidogrel b. Warfarin (Coumadin) c. Carotid endarterectomy d. Carotid artery stenting e. Tissue plasminogen activator

c. Carotid endarterectomy

A 31-year-old man with a history of diabetes mellitus and sickle cell anemia presents to the emergency department with 2 days of cough, chest pain, and difficulty breathing. His temperature is 100.7 °F, blood pressure is 112/65 mm Hg, heart rate is 117 bpm, respiratory rate is 22 breaths/min, and pulse oximetry shows 94% on room air. White blood cell count is 13,000/mm3, and hemoglobin is 9.9 g/dL (baseline of 10.1 g/dL). Chest x-ray reveals a right lower lobe opacity. Blood and sputum cultures are collected. Which of the following is the most appropriate empiric antibiotic therapy for this patient? a. Vancomycin b. Ceftriaxone c. Cefotaxime and azithromycin d. Penicillin

c. Cefotaxime and azithromycin

A 68-year-old man was hospitalized with persistent fever and a new heart murmur and diagnosed with Streptococcus bovis endocarditis of the mitral valve. After receiving 10 days of intravenous antimicrobial therapy, he is noted to be afebrile and with symptoms resolved. At this time, which of the following is the most important next step? a. Good dental hygiene and proper denture fitting to prevent reinfection of damaged heart valves from oral flora. b. Repeat echocardiography in 6 weeks to ensure the vegetations have resolved. c. Colonoscopy to look for mucosal lesions. d. Mitral valve replacement to prevent systemic emboli such as cerebral infarction.

c. Colonoscopy to look for mucosal lesions.

A 59-year-old man with a known history of COPD presents with worsening dyspnea. On examination, he is afebrile. His breath sounds are decreased bilaterally. He is noted to have jugular venous distension (JVD) and 2+ pitting edema of the lower extremities. Which of the following is the most likely cause of his increasing dyspnea? a. COPD exacerbation b. Pneumonia c. Cor pulmonale d. Pneumothorax

c. Cor pulmonale

A 49-year-old sedentary man has made an appointment because his best friend died of a myocardial infarction at age 50. He asks your advice about exercise and weight loss to prevent cardiovascular disease. In counseling him, which of the following statements regarding exercise is most accurate? a. To be beneficial, exercise must be performed for at least 15 minutes every day. b. Walking for exercise has not been shown to improve meaningful clinical outcomes. c. Counseling patients has not been shown to increase the number of patients who exercise. d. Intense exercise offers no health benefit over mild-to-moderate amounts of exercise.

c. Counseling patients has not been shown to increase the number of patients who exercise.

A 33-year-old man is complaining of the room spinning. Which of the following tests is used to diagnose BPPV? a. Weber test b. Rinne test c. Dix-Hallpike maneuver d. Brandt-Daroff maneuver e. Epley maneuver

c. Dix-Hallpike maneuver

A 78-year-old woman is being followed in the office for cognitive decline over the past year. The patient's daughter states that the patient has been forgetting where she is going and has left the stove on for hours. She is otherwise healthy and denies neurologic symptoms. A workup including laboratory work and head imaging has been negative. The patient is diagnosed with early AD. Which of the following agents is most likely to help with the cognitive function? a. Haloperidol b. Estrogen replacement therapy c. Donepezil d. High-dose vitamin B12 injections

c. Donepezil

A 39-year-old man develops a moderate free-flowing pleural effusion following a left lower lobe pneumonia. Thoracentesis reveals straw-colored fluid with gram-positive diplococci on Gram stain, pH 6.9, glucose 32 mg/dL, and LDH 1890. Which of the following is the best next step? a. Send the fluid for culture. b. Continue treatment with antibiotics for pneumococcal infection. c. Drain the effusion via tube thoracostomy. d. Schedule a follow-up chest x-ray in 2 weeks to document resolution of the effusion.

c. Drain the effusion via tube thoracostomy.

An 8-year-old boy is brought into the pediatrician's office for fatigue, pain of the joints, and red-brown colored urine of 2 days. On examination, the blood pressure is 140/92 mm Hg, and heart rate is 90 beats per minute. He has facial swelling and pedal edema. The heart, lung, and abdominal examinations are normal. His mother states that about 3 weeks ago he had a sore throat and fever. Which of the following laboratory findings would most likely be present? a. Elevated serum complement levels b. Positive antinuclear antibody titers c. Elevated ASO titers d. Positive blood cultures e. Positive cryoglobulin titers

c. Elevated ASO titers

A 38-year-old woman is being seen by her primary care provider for a regular follow-up. The patient's BMI is 34 kg/m2 with significant abdominal adiposity. The patient has hypothyroidism, hypertension, diabetes, and hyperlipidemia, which are well controlled with levothyroxine, lisinopril, metformin, and atorvastatin. She works as an executive assistant. She has never smoked cigarettes and does not drink alcohol. What is the next best step in management of the patient's metabolic syndrome? a. Recheck HbA1c, fasting lipid panel, triiodothyronine (T3), and thyroid-stimulating hormone (TSH) b. Refer the patient for bariatric surgery c. Encourage the patient to perform moderate-intensity physical exercise d. Obtain an electrocardiogram (ECG)

c. Encourage the patient to perform moderate-intensity physical exercise

A 40-year-old man travels to South America and develops watery diarrhea 1 day after coming back to the United States. He had been in good health previously. Which of the following is the most likely etiology of the symptoms? a. Rotavirus b. Giardia c. Escherichia coli d. Staphylococcus aureus e. Cryptosporidium

c. Escherichia coli

A 70-year-old woman presents for evaluation of a lesion on her left cheek. It has been present for several months. It is slowly enlarging and bleeds if she scratches it. On examination, you find a 7-mm diameter, pearly appearing papule with visible telangiectasias on the surface. Which of the following is the appropriate management of this lesion? a. Close observation and reexamination in 3 months b. Reassurance of the benign nature of the lesion c. Excision d. Local destruction by freezing with liquid nitrogen

c. Excision

A 39-year-old man is noted to have a DVT without any known risk factors. He notes that his brother also developed a PE at age 45, and his mother developed a "clot in the leg" when she was in her 30s. Which of the following is the most likely inherited disorder in this patient? a. Protein S deficiency b. Antithrombin III deficiency c. Factor V Leiden mutation d. Antiphospholipid antibody syndrome e. Familial malignancy syndrome

c. Factor V Leidien mutation

A 34-year-old woman is being seen in the urgent care center for "feeling ill." She was diagnosed with hyperthyroidism 6 months ago, but she has not taken her medications for 5 days. After the history and physical examination, the provider is suspicious of possible thyroid storm. Which of the following features would best distinguish hyperthyroidism from thyroid storm? a. Heart rate of 120 bpm b. Weight loss c. Fever and delirium d. Large goiter

c. Fever and delirium

Which of the following is the most likely explanation for multiple drug resistance to antibiotics that spreads from one type of bacteria to another? a. Adaptation b. Decreased bioavailability c. Gene transfer d. Mutation

c. Gene transfer

A 39-year-old woman with hypertension and type 2 diabetes has been noted to have progressively worsening renal insufficiency. Which of the following measures is most important in the prevention of ESRD? a. Tobacco cessation b. Triglyceride control c. Glycemic control d. Weight control e. Dietary sodium restriction

c. Glycemic control

A 44-year-old woman is being seen in the office for a 2-month history of progressive anxiety, nervousness, and tremor. She also says she has heat intolerance. On exam, her HR is 110 bpm and BP is 130/80 mm Hg. Her thyroid gland is diffusely enlarged and nontender, and an audible bruit is present. Her serum TSH level is 0.01 mIU/L (normal 0.35-5.0). Which of the following is the most likely diagnosis? a. Lymphocytic thyroiditis b. Hashimoto thyroiditis c. Graves disease d. Multinodular toxic goiter

c. Graves disease

A 65-year-old man with a history of prosthetic aortic valve, hypertension, and osteoarthritis is hospitalized for an elective knee replacement surgery. The patient is a nonsmoker but drinks one or two glasses of wine on the weekends. His admission CBC shows a WBC count of 8000/mm3, hemoglobin of 9.2 g/dL, and a platelet count of 250,000/mm3. Medications started on admission include acetaminophen, heparin prophylaxis, lisinopril, and pantoprazole. The patient's hospital course was complicated by acute kidney injury and postsurgery ileus. Five days after his knee surgery, laboratory tests are significant for a platelet count of 62,000/mm3. Which of the following is the most likely cause of the thrombocytopenia? a. Acetaminophen b. Alcohol intake c. Heparin d. Prosthetic heart valve

c. Heparin

A 36-year-old man has been recently diagnosed with type 2 diabetes. If not vaccinated previously, which of the following immunizations is most important to administer? a. Toxoplasmosis b. Human papilloma virus (HPV) c. Hepatitis B d. Rubella

c. Hepatitis B

Two weeks after a viral syndrome, a 2 year old develops bruising and generalized petechiae that is more prominent over the legs. He has neither hepatosplenomegaly nor lymph node enlargement. Laboratory testing reveals a normal hemoglobin, hematocrit, and white blood cell count and differential. The platelet count is 15,000/mm3. Which of the following is the most likely diagnosis? a. Acute lymphoblastic leukemia b. Aplastic anemia c. Immune thrombocytopenic purpura d. Thrombotic thrombocytopenic purpura e. von Willebrand disease

c. Immune thrombocytopenic purpura

A 21-year-old woman presents for her first Pap smear. She completed the HPV vaccine series by age 19. Assuming that her examination and Pap smear results are normal, when would you recommend that she return for a follow-up Pap smear? a. In 6 months, as the first Pap smear should be repeated within a year to reduce a potential false-negative result b. In 1 year, as she is higher risk of cervical cancer because of her age c. In 3 years, as the Pap smear was normal d. In 5 years, as she is at low risk because she received the HPV vaccine

c. In 3years, as the Pap smear was normal

A 68-year-old patient with known COPD has been having frequent exacerbations of his COPD. Pulmonary function testing shows a FEV1 of 40% predicted (normal = 80% to 120%). His SaO2 by pulse oximetry is 91%. Which of the following medication regimens is the most appropriate at this time? a. Inhaled salmeterol twice daily and albuterol as needed b. Oral albuterol daily and inhaled fluticasone twice daily c. Inhaled fluticasone twice daily, inhaled tiotropium twice daily, and inhaled albuterol as needed d. Inhaled fluticasone twice daily, inhaled tiotropium twice daily, inhaled albuterol as needed, and oxygen therapy

c. Inhaled fluticasone twice daily, inhaled tiotropium twice daily, and inhaled albuterol as needed

An obese 50-year-old man with a history of asthma is being seen in the clinic with complaints of occasional dyspepsia and nocturnal cough. He notes that he wakes up in the morning with a sour taste in his mouth. His current medications include an inhaled corticosteroid and a short-acting beta-2-agonist. Which of the following should be your next step? a. 24-hour esophageal pH monitoring b. Chest radiograph c. Initiation of omeprazole d. Short course of oral corticosteroids e. Initiation of allergy desensitization

c. Initiation of omeprazole

Which of the following is the best treatment for a 39-year-old woman with fever of 103 °F, nausea, flank pain, and more than 105 CFU/mL of E. coli in a urine culture? a. Oral trimethoprim-sulfamethoxazole for 3 days b. Single-dose ciprofloxacin c. Intravenous and then oral levofloxacin for 14 days d. Oral ampicillin for 21 to 28 days

c. Intravenous and then oral levofloxacin for 14 days

An 80-year-old woman is brought to the emergency department with altered mental status and fever. She is awake and cooperative but is not oriented to time or place. Her blood pressure is normal, her pulse is normal, and her temperature is 101 °F. She is found to have pneumonia. Laboratory testing reveals a sodium level of 130 mEq/L but otherwise normal electrolytes. Which of the following is the most appropriate treatment? a. Intravenous antibiotic only b. Intravenous antibiotic and aggressive rehydration with intravenous normal saline c. Intravenous antibiotic and fluid restriction d. Intravenous antibiotic and intravenous 3% saline

c. Intravenous antibiotic and fluid restriction

Which of the following ECG changes makes the determination of acute MI the most difficult? a. Q wave b. ST-segment elevation c. Left bundle branch block d. First-degree atrioventricular block e. T-wave inversion

c. Left bundle branch block

A 73-year-old woman presents to the office due to concern about several tan-colored moles on her arms, face, and ears that have progressively grown over the past 6 months. Upon further examination, the moles are determined to be between 6 and 8 mm with very irregular borders. The clinician decides to obtain an excisional biopsy. Which of the following skin lesions should the provider be most suspicious of based on the history and physical examination? a. Benign nevus b. Superficial spreading melanoma c. Lentigo maligna melanoma d. Acral lentiginous melanoma

c. Lentigo maligna melanoma

A mother brings her 3-year-old son with Down syndrome to the clinic because his gums have been bleeding for 1 week. She reports that he has been less energetic than usual. Examination reveals that the child has an oral temperature of 100°F (37.8°C), pallor, splenomegaly, gingival bleeding, and bruises on the lower extremities. Which of the following is most likely? a. Aplastic anemia b. Idiopathic thrombocytopenic purpura (ITP) c. Leukemia d. Leukemoid reaction e. Megaloblastic anemia

c. Leukemia

An 18-year-old man is being seen the emergency center for nausea, vomiting, lightheadedness, and fatigue. He is a known type 1 diabetic and states that he has been taking his insulin as scheduled. On examination, his BP is 80/40 mm Hg, temperature is 101 °F, HR is 120 bpm, and RR is 30 breaths/min. He has some adenopathy of the cervical area. Laboratory test values show an arterial pH of 7.20, po2 of 100 mm Hg, pco2 of 28 mm Hg, and HCO3 of 12 mEq/L. His serum glucose level is 400 mg/dL. Which of the following is the most accurate statement regarding this patient's likely potassium status? a. Likely to have a serum potassium level less than 3 mEq/L. b. Likely to have a serum potassium level more than 7 Eq/L. c. Likely to have a total body potassium deficit regardless of the serum level. d. Serum level is likely to increase with correlation of the acidosis.

c. Likely to have a total body potassium deficit regardless of the serum level.

A 21-year-old man with known asthma has been placed on a regimen consisting of inhaled corticosteroids and intermittent (short-acting) beta-2-agonist. He is being seen in the office with a new complaint of nocturnal awakenings secondary to cough and occasional wheezing. These episodes occur three to four times per week. Six months ago, his pulmonary function testing revealed an FEV1 of 80% of predicted, and FEV1/FVC of 70% of predicted. Which of the following is the best next step? a. Oral steroids b. Leukotriene inhibitors c. Long-acting beta-2 agonists (LABAs) d. Theophylline e. Antireflux therapy

c. Long-acting beta-2 agonists (LABAs)

In the patient described in question 10.5, which of the following is the treatment of choice for his diarrhea (Clostridium difficile)? a. Ciprofloxacin b. Azithromycin c. Metronidazole d. Loperamide

c. Metronidazole

Which of the following statements regarding breast cancer screening is true? a. Breast self-examinations have been shown to decrease mortality rates b. Clinical breast examinations in conjunction with routine mammography have been shown to improve mortality rates c. Most abnormalities found on routine mammography are not breast cancer d. Since breast cancer rates increase in older women, there is no upper age at which breast cancer screening may be discontinued

c. Most abnormalities found on routine mammography are not breast cancer

An 18-year-old marathon runner has been training during the summer. He is brought to the emergency department disoriented after collapsing on the track. His temperature is 102 °F. A Foley catheter is placed and reveals reddish urine with 3+ blood on dipstick and no cells seen microscopically. Which of the following is the most likely explanation for his urine? a. Underlying renal disease b. Prerenal azotemia c. Myoglobinuria d. Glomerulonephritis

c. Myoglobinuria

A 30-year-old man has both mild persistent asthma and chronic environmental allergies. Which of the following medications is indicated for the management of this patient's conditions? a. Inhaled albuterol (short-acting beta-adrenergic agonist) b. Intranasal fluticasone (corticosteroid) c. Oral montelukast (leukotriene modifier) d. Oral cetirizine (second-generation antihistamine)

c. Oral montelukast (leukotriene modifier)

Which of the following is the most likely organism responsible in Question 58.3 (Aplastic crisis)? a. Streptococcus pneumoniae b. Salmonella spp c. Parvovirus B19 d. Staphylococcus aureus

c. Parvovirus B19

The patient described in Question 10.1 is hospitalized, but there is a delay in initiating treatment. You are called to the bedside because he has become hypotensive with a systolic blood pressure of 85/68 mm Hg, a heart rate of 122 bpm, and pulsus paradoxus. A repeat ECG is unchanged from admission. Which of the following is the most appropriate immediate intervention? a. Draw blood cultures and initiate broad-spectrum antibiotics for suspected sepsis. b. Give intravenous furosemide for fluid overload. c. Perform echocardiographic-guided pericardiocentesis. d. Perform percutaneous coronary intervention for acute MI.

c. Perform echocardiographic-guided pericardiocentesis.

A 30-year-old woman with ITP comes to her outpatient hematology office for a routine follow-up. She reports that despite taking maximum corticosteroid doses, she still has a platelet count of 20,000/mm3 and frequent bleeding episodes. Which of the following should she receive before her splenectomy? a. Bone marrow radiotherapy b. Intravenous interferon therapy c. Pneumococcal vaccine d. Washed leukocyte transfusion

c. Pneumococcal vaccine

A 70-year-old man is having difficulty hearing his family members' conversations. He is diagnosed with presbycusis. Which of the following statements regarding his condition is most accurate? a. Presbycusis does not respond to hearing aid use. b. Presbycusis is usually caused by a conductive disorder. c. Presbycusis usually results in loss of speech discrimination. d. Presbycusis usually results in unilateral hearing loss. e. Presbycusis usually results in low-frequency hearing loss.

c. Presbycusis usually results in loss of speech discrimination.

A 58-year-old woman is being seen in the office for her first evaluation of hyperthyroidism. On examination, she is found to have a diffusely slightly enlarged thyroid gland that is nontender. She has exophthalmos, tremor, and brisk deep tendon reflexes. Serum TSH is 0.03 mIU/L (normal 0.35-5.0). Which of the following is the best therapy for this patient? a. Long-term oral propranolol b. Lifelong oral PTU c. Radioactive iodine ablation d. Surgical thyroidectomy

c. Radioactive iodine ablation

A 68-year-old white female presents to the ED with palpitations that started several hours ago. On physical examination she has an irregular heart beat with a rate of 75 to 90 bpm and a thready pulse. ECG shows atrial fibrillation. Her past medical history is significant for Type 2 diabetes diagnosed 10 years ago and a cholecystectomy last year. What therapy improves survival in patients with chronic atrial fibrillation? a. Pharmacologic cardioversion b. Rate control with nodal blocking agents c. Rhythm control with amiodarone d. Electrical cardioversion

c. Rhythm control with amiodarone

A 25-year-old woman complains of pain in her PIP and metacarpophalangeal joints and reports a recent positive antinuclear antibody (ANA) laboratory test. Which of the following clinical features would not be consistent with a diagnosis of SLE? a. Pleural effusion b. Malar rash c. Sclerodactyly d. Urinary sediment with red blood cell casts

c. Sclerodactyly

A 50-year-old man has been treated for rheumatoid arthritis for many years. He currently is taking corticosteroids for the disease. On examination, he has stigmata of rheumatoid arthritis and some fullness on his left upper abdomen. His platelet count is slightly low at 105,000/mm3. His WBC count is 3100/mm3 with neutropenia, and hemoglobin level is 9 g/dL. Which of the following is the most likely etiology of the thrombocytopenia? a. Autoimmune destruction b. Prior gold therapy c. Splenic sequestration d. Steroid induced

c. Splenic sequestration

An 82-year-old woman is admitted to the hospital for altered mental status. Her family says that she has been confused, falling asleep frequently, and hallucinating—talking to people who are not in the room. Her family reports that prior to this illness, the patient was independent and "sharp as a tack." On urine analysis, she is found to have a urinary tract infection. Which of the following is the most appropriate treatment? a. Start rivastigmine (Exelon) for worsening Alzheimer dementia. b. Start an alerting agent such as modafinil (Provigil) for symptomatic treatment for her hypersomnia. c. Start an antibiotic for treatment of her infection and optimize management of any other medical conditions. d. Giver her a dose of ziprasidone (Geodon) for her hallucinations.

c. Start an antibiotic for treatment of her infection and optimize management of any other medical conditions.

A 55-year-old man is presenting for an annual examination. The patient's BMI is 24 kg/m2. His blood pressure is 123/77 mm Hg with a heart rate of 71 bpm. The patient's total cholesterol is 171 mg/dL, HDL is 45 mg/dL, and LDL is 90 mg/dL. Last year, his Hb A1C was 5.2% and 10-year ASCVD risk score was 4.8%. Laboratory tests drawn today are unremarkable except for elevated Hb A1C at 7%. He currently takes no medications. He does not smoke or drink. What is the next best step in management? a. Start statin therapy b. Start metformin c. Start statin and metformin therapy d. No further changes necessary

c. Start statin and metformin therapy

A 62-year-old man who works at an automobile assembly line has noticed that he feels pain, fatigue, and numbness in his right arm while working for the last several months. This morning at work, he noticed vertigo, then light-headedness, then lost consciousness for a few seconds. The BP in his right arm is 30 mm Hg lower than that in his left arm. What is the most likely diagnosis? a. Left middle cerebral artery stroke b. Lacunar infarction involving right internal capsule c. Stenosis of right subclavian artery due to atherosclerosis d. Multiple sclerosis

c. Stenosis of right subclavian artery due to atherosclerosis

A 35-year-old woman presents with calf tenderness and acute dyspnea. The arterial blood gas reveals a partial pressure of oxygen (Po2) of 76 mm Hg. Which of the following is the most common physical examination finding of PE? a. Wheezing b. Increased pulmonary component of the second heart sound c. Tachypnea d. Calf swelling e. Pulmonary rales

c. Tachypnea

At a routine checkup, a 6-year-old boy is found to have a BP of 150/90 mm Hg. Repeated BP readings are consistently elevated. The child was delivered at 36 weeks by normal spontaneous vaginal delivery with no complications. All major milestones were met on time, and he currently is enrolled in first grade. The child has been healthy up until this point. Which of the following are the most appropriate diagnosis and therapeutic step? a. The child has essential hypertension and should be started on the DASH diet. b. The child most likely has hyperthyroidism, should be started on a beta-blocker, and should have thyroid studies performed. c. The child most likely has renal parenchymal disease and should have a urinalysis and renal ultrasound ordered. d. The child most likely has "white coat" hypertension; follow up in 3 months if there is no family history of hypertension. e. The child most likely has a pheochromocytoma and should start a 24-hour urine collection for metanephrines.

c. The child most likely has renal parenchymal disease and should have a urinalysis and renal ultrasound ordered.

A 59-year-old man with a long history of diabetes with chronic renal insufficiency due to diabetic nephropathy is seen in clinic for routine laboratory work. He is asymptomatic, but his glucose is elevated at 258 mg/dL. His other chemistries are as follows: sodium 135 mEq/L, potassium 5.4 mEq/L, chloride 108 mEq/L, and bicarbonate 18 mEq/L. His creatinine is stable at 2.1 mg/dL. Which of the following is the most likely cause of this patient's acidemia? a. Diabetic ketoacidosis b. Lactic acidosis c. Type 4 renal tubular acidosis (RTA) d. Accidental salicylate overdose

c. Type 4 renal tubular acidosis (RTA)

A 75-year-old man is noted to have chest pain with exertion and has been passing out recently. On examination, he is noted to have a harsh systolic murmur. Which of the following is the best therapy for his condition? a. Coronary artery bypass b. Angioplasty c. Valve replacement d. Carotid endarterectomy

c. Valve replacement

A 75-year-old woman with type 2 diabetes for approximately 20 years, diabetic retinopathy, and diabetic nephropathy is brought into the clinic by her daughter for follow-up. The patient's last serum creatinine level was 2.2 mg/dL. The patient currently takes a glyburide for her diabetes and lisinopril for her proteinuria. Her daughter reports that on three occasions in the past 2 weeks, her mother became sweaty, shaky, and confused, which resolved when she was given some orange juice. Which of the following conditions is most likely to be contributing to these episodes? a. Excess caloric oral intake b. Interaction between the ACE inhibitor and the sulfonylurea agents c. Worsening renal function d. Hyperglycemic amnesia

c. Worsening renal function

A 1-year-old boy presents with fever, ear pain, and purulent discharge from both eyes. On examination, bilateral erythematous, bulging tympanic membranes (TMs), and purulent conjunctivitis are noted. Based on his symptoms and examination, amoxicillin-clavulanate is prescribed. The mother asks what she could do to prevent future ear infections. Which of the following is NOT one of the current recommendations to reduce the incidence of otitis media? a. Pneumococcal vaccine b. Influenza vaccine c. Xylitol d. Eliminating exposure to tobacco smoke e. Breast-feeding

c. Xylitol

Which one of the following patients presenting with chest pain is at the highest risk for an acute MI? a. A 40-year-old woman on a proton pump inhibitor for reflux disease b. A 75-year-old man with parasternal chest pain, lipid abnormalities, and no past history or cardiac disease c. A 23-year-old man recently diagnosed with hypertrophic cardiomyopathy d. A 67-year-old man with a history of a prior angioplasty, with chest pain radiating to the neck and complaint of diaphoresis

d. A 67-year-old man with a history of a prior angioplasty, with chest pain radiating to the neck and complaint of diaphoresis

A 6-year-old boy with a history of sickle cell anemia is brought to the emergency department by his parents because of the boy's 4 days of fatigue. The patient is up to date on all his immunizations. His home medications include penicillin and folic acid. His temperature is 99.3 °F, blood pressure is 102/70 mm Hg, heart rate is 116 bpm, respiratory rate is 22 breaths/min, and oxygen saturation shows 96% on room air. On physical examination, he appears pale, but he has normal chest, respiratory, and abdominal examinations. His white blood cell count is 7000/mm3, hemoglobin is 6.5 g/dL, platelet count is 27,000/mm3, and reticulocyte count is 0.2%. Which of the following is the most likely diagnosis? a. Acute chest syndrome b. Splenic sequestration c. Aplastic anemia d. Aplastic crisis

d. Aplastic crisis

Which of the following counseling strategies is most likely to enhance your patients' smoking cessation rates? a. Discuss smoking cessation techniques only with patients who ask for your advice, as others will resent your suggestions. b. Emphasize primarily the health risks of smoking. c. Note in each patient's chart that you have discussed cessation, so that you do not repeat the message to the same patient at subsequent visits. d. Ask about smoking cessation at each encounter.

d. Ask about smoking cessation at each encounter.

A 51-year-old man is being seen for an annual physical examination. The patient's BMI is 28 kg/m2, blood pressure is 141/72 mm Hg, and heart rate is 73 bpm. Laboratory tests showed normal complete blood count (CBC), normal basic metabolic panel (BMP), Hb A1C of 7%, and total cholesterol of 245 mg/dL. The patient has smoked 1 pack per day for the last 30 years. He exercises 30 minutes daily. What is the best step in management of this patient? a. Calculate the patient's ASCVD risk score b. Initiate statin therapy c. Initiate metformin therapy d. Assess readiness for smoking cessation e. Schedule an appointment for a blood pressure check in 1 week

d. Assess readiness for smoking cessation

Ms N is a widowed 80-year-old woman who comes to your office for a yearly physical examination. She is doing well and has no complaints. A routine ECG is performed and reveals atrial fibrillation with a heart rate of 75 bpm. Blood pressure is similar to her last visit, which is 128/76 mm Hg. She does not have a history of diabetes, heart failure, or stroke. Her current medications include mirtazapine for treatment of depression. What is the appropriate management of this patient? a. Recommend one 325mg aspirin per day for antithrombic therapy. b. Repeat ECG in 3 months. If atrial fibrillation is still present, begin adjusted-dose warfarin therapy. c. Immediately begin low molecular weight heparin injections and start warfarin therapy. d. Begin adjusted dose warfarin therapy.

d. Begin adjusted dose warfarin therapy.

A 49-year-old man with a long-standing history of chronic renal failure as a consequence of diabetic nephropathy is brought to the emergency room for nausea, lethargy, and confusion. His physical examination is significant for an elevated jugular venous pressure, clear lung fields, and harsh systolic and diastolic sounds heard over the precordium. Serum chemistries reveal K 5.1 mEq/L, CO2 17 mEq/L, BUN 145 mg/dL, and creatinine 9.8 mg/dL. Which of the following is the most appropriate next step in therapy? a. Administer intravenous insulin and glucose b. Administer intravenous sodium bicarbonate c. Administer intravenous furosemide d. Begin hemodialysis urgently

d. Begin hemodialysis urgently

Penicillins inhibit which of the following bacterial processes/compounds? a. Protein synthesis b. Topoisomerase c. Dihydropteroate synthase d. Cell-wall synthesis

d. Cell-wall synthesis

A 66-year-old Caucasian woman has an average BP of 155/70 mm Hg despite appropriate lifestyle modification efforts. Her only other medical problems are osteopenia, kidney stones, and mild depression. Her last lipid panel revealed a total cholesterol of 160 mg/dL, high-density lipoprotein of 40 mg/dL, and low-density lipoprotein of 90 mg/dL. Which of the following would be the most appropriate treatment at this time? a. Lisinopril (Prinivil, Zestril) b. Propranolol (Inderal) c. Amlodipine (Norvasc) d. Chlorthalidone e. Losartan (Cozaar)

d. Chlorthalidone

A 45-year-old man presents with 3 days of watery diarrhea and abdominal cramping. He has no sick contacts and has not traveled recently. He is not currently taking any medications, but he was prescribed amoxicillin 2 weeks ago for a sinus infection. Which of the following tests is most likely to identify the cause of his diarrhea? a. Stool guaiac b. Evaluation of stool for fecal leukocytes c. Evaluation of stool for ova and parasites d. Clostridium difficile toxin immunoassay

d. Clostridium difficile toxin immunoassay

A 52-year-old man comes into the outpatient clinic for an annual "checkup." He is in good health and has a relatively unremarkable family history. He has never smoked cigarettes. For which of the following disorders should a screening test be performed? a. Prostate cancer b. Lung cancer c. Abdominal aortic aneurysm d. Colorectal cancer e. Skin cancer

d. Colorectal cancer

A 34-year-old woman of Mediterranean descent is found to have mild anemia on prenatal screening CBC. She denies any shortness of breath, fatigue, or blood in stool. Family history is notable for a brother who requires frequent blood transfusions. A CBC demonstrates microcytic, hypochromic anemia with high serum iron, normal RDW, normal ferritin, and low reticulocytes. Hemoglobin electrophoresis is normal. Which of the following is the best next diagnostic step? a. Check lead levels b. Repeat hemoglobin electrophoresis for improved sensitivity c. Bone marrow biopsy d. DNA testing e. Start ferrous sulfate 325mg daily and recheck CBC in 6 weeks

d. DNA testing

A 56-year-old woman is being seen in the office for a history of shortness of breath that has worsened over the past month. She admits to a 60 pack-year smoking history. She complains of fatigue and dyspnea with minimal exertion and a productive cough each morning. Pulmonary function testing is ordered. Which of the following is the most likely finding in this patient? a. Higher diffusing capacity of lung for carbon monoxide (DLCO) b. Decreased residual volume c. Normal FEV1 d. Decreased FEV1/FVC e. Decreased FVC

d. Decreased FEV1/FVC

A 67-year-old man presents with dizziness and a positive stool guaiac test. a. Normal MMA; decreased serum folate level b. Elevated MMA; decreased serum B12 level c. Elevated ferritin; normal MCV; decreased serum iron level d. Decreased ferritin; decreased MCV; decreased serum iron level

d. Decreased ferritin; decreased MCV; decreased serum iron level

A 43-year-old man is admitted to the hospital with a diagnosis of acute pancreatitis. His family shares that he is a heavy user of alcohol. He is given intravenous hydration and is placed NPO. Which of the following findings is the highest predictor of mortality? a. His age b. Initial serum glucose level of 60 mg/dL c. BUN of 18 mg/dL d. Disorientation, with Glasgow Coma Scale score of 10 e. Amylase level of 1000 IU/L

d. Disorientation, with Glasgow Coma Scale of 10

A 58-year-old man presents to his provider for follow-up of his hypertension and hyperlipidemia. He also reports chest pain and feeling short of breath after climbing two flights of stairs or walking three to four blocks. The symptoms resolve after several minutes of rest. Which of the following drugs would not be indicated as a first-line agent in the treatment of this patient's condition? a. Atorvastatin b. Nitroglycerin c. Enalapril d. Doxazosin e. Aspirin

d. Doxazosin

Which of the following tests is necessary to make the diagnosis of acute leukemia? a. Chromosomal analysis of cells obtained from a bone marrow biopsy b. Chromosomal analysis of cells obtained from a lymph node biopsy c. Cytochemical staining of cells obtained from a bone marrow biopsy d. Examination of smears made from a bone marrow aspiration e. Examination of smears made from the peripheral blood

d. Examination of smears made from a bone marrow aspiration

A 56-year-old man presents to his provider with symptoms consistent with allergic rhinitis. His past medical history is positive for benign prostatic hyperplasia. He continues to work in a warehouse as a forklift operator. Which of the following medications should be used to treat this patient? a. Diphenhydramine b. Hydroxyzine c. Chlorpheniramine d. Fexofenadine

d. Fexofenadine

A 22-year-old man complains of acute hemoptysis over the past week. He denies smoking, fever, or preexisting lung disease. His blood pressure is 130/70 mm Hg, and his physical examination, including lung examination, is normal. His urinalysis shows microscopic hematuria and RBC casts. Which of the following is the most likely etiology? a. Metastatic renal cell carcinoma to the lungs b. Acute tuberculosis of the kidneys and lungs c. Systemic lupus erythematosus d. Goodpasture disease (antiglomerular basement membrane)

d. Goodpasture disease (antiglomerular basement membrane)

An 18-year-old young man is noted to have motor tics and involuntary, obscene vocalizations. Which of the following medications is indicated in the treatment of this disorder? a. Trihexyphenidyl b. Phenytoin c. Carbamazepine d. Haloperidol e. Levodopa

d. Haloperidol

A third-year medical student is researching various recommendations for the care of the geriatric patient. Which of the following statements is most accurate? a. The USPSTF recommends routine screening for colorectal cancer for adult men starting at the age of 50 and for women starting at the age of 65. b. The USPSTF recommends continuing screening for cervical cancer with Pap smear in all women until the age of 75. c. The USPSTF recommends that all men should be screened for prostate cancer with prostate antigen (PSA) testing annually starting at the age of 50. d. Herpes zoster vaccination is recommended for all adults over the age of 50.

d. Herpes zoster vaccination is recommended for all adults over the age of 50.

A 59-year-old woman has been placed on warfarin after being found to have chronic AF. She is noted to have an INR of 5.8, is asymptomatic, and has no overt bleeding. Which of the following is the best management for this patient? a. Transfuse with erythrocytes. b. Give vitamin K. c. Give fresh frozen plasma d. Hold warfarin.

d. Hold warfarin.

A 12-year-old girl is brought to the emergency department because of a severe sore throat, muffled voice, drooling, and fatigue. She has been sick for the past 3 days and is unable to eat because of painful swallowing. The parents deny any history of recurrent pharyngitis. The patient still manages to open her mouth, and you are able to see an abscess at the upper pole of the right tonsil with deviation of the uvula toward the midline. Examination of the neck reveals enlarged and tender lymph nodes. Which of the following is the most appropriate management? a. Analgesics for pain b. Oral antibiotics c. Nebulized racemic epinephrine d. Incision and drainage of the abscess Tonsillectomy and adenoidectomy

d. Incision and drainage of the abscess

A 45-year-old man is brought into the emergency center for severe abdominal pain and light-headedness. His wife states that the patient has had lower abdominal pain for 2 days but did not want to see a doctor. He is noted to have a blood pressure of 80/40 mm Hg, heart rate of 142 bpm, and temperature of 102 °F. His abdomen is tender with guarding and rebound, particularly in the right lower quadrant. Acute appendicitis is diagnosed. Three liters of 0.9% saline are infused, and IV antibiotics are administered as he is prepared for surgery. After the saline, his blood pressure is 70/42 mm Hg. Which of the following is the most appropriate next step? a. Administer a beta-blocker to control his heart rate. b. Check a cortisol level and administer corticosteroids. c. Infuse fresh frozen plasma (FFP). d. Initiate norepinephrine IV infusion. e. Initiate IV morphine for pain control.

d. Initiate norepinephrine IV infusion.

A 66-year-old man with known metastatic squamous cell carcinoma of the esophagus is brought to the emergency department for increasing lethargy and confusion. He is clinically dehydrated, his serum calcium level is 14 mg/dL, and his creatinine level is 2.5 mg/dL, though 1 month ago it was 0.9 mg/dL. Which therapy for his hypercalcemia should be instituted first? a. Intravenous bisphosphonate b. Intravenous furosemide c. Glucocorticoids d. Intravenous normal saline e. Chemotherapy for squamous cell carcinoma

d. Intravenous normal saline

A 9-year-old girl is being seen in your office with fever and difficulty breathing. You are concerned about the diagnosis of epiglottitis. Which of the following is the most accurate statement regarding epiglottitis? a. The child usually prefers to be in a prone position. b. The radiographic finding of steeple sign will be present. c. Every effort should be made to visualize the epiglottis in the office to confirm the diagnosis. d. Its diagnosis is decreasing in incidence.

d. Its diagnosis is decreasing in incidence.

A 65-year-old man with a medical history of uncontrolled hypertension, mild biventricular systolic heart failure, and a 40 pack-year smoking history presents to the emergency room with 1 week of worsening cough, fever, and dyspnea at rest. His symptoms also include diffuse myalgia, abdominal pain, nonbloody diarrhea, and a rapidly worsening nonproductive cough. He denies alcohol or drug history and endorses being married to his wife for 40 years. On admission, his vital signs are: temperature of 38 °C, blood pressure (BP) 160/82 mm Hg, heart rate 89 beats per minute (bpm), respiratory rate (RR) 25 breaths/min, and SpO2 (oxygen saturation as measured by pulse oximetry) is 94% on room air. Which of the most likely organisms is the etiology of his illness? a. Aspergillus fumigatus b. Chlamydia pneumoniae c. Coccidioidomycosis d. Legionella pneumophila e. Mycoplasma pneumoniae

d. Legionella pneumophila

A 48-year-old man presents for follow-up of an elevated calcium level of 12.3 mg/dL found on routine screening laboratory tests at his last health maintenance visit. He takes no medications other than an occasional antihistamine for allergies. He recently started smoking a half-pack of cigarettes per day. He was prompted to attend to today's health maintenance visit by his wife, who claims that he has become forgetful, has a decreased appetite, and has had a 10-lb weight loss over the past 2 months. As part of his follow-up laboratory tests, you obtain a serum PTH, which comes back within the normal range. Which of the following is the next step in diagnosis? a. Chest x-ray b. Repeat calcium after hydration c. Measurement of PTH-rP levels d. Measurement of urinary calcium excretion

d. Measurement of urinary calcium excretion

Which of the following is most accurate? a. Cough caused by captopril may resolve with switching to enalapril. b. Initial treatment of a chronic cough should include codeine or a similar opiate derivative to suppress the cough. c. Cough caused by reflux can be effectively ruled out by a negative history of heartburn or dyspepsia. d. More than one condition is often responsible for causing chronic cough in a given patient.

d. More than one condition is often responsible for causing chronic cough in a given patient.

One year ago, a 24-year-old woman had an episode of diplopia of 2 weeks' duration. The symptoms resolved completely. Currently, she complains of left arm weakness but no headache. Which of the following is the most likely diagnosis? a. Recurrent TIAs b. Subarachnoid hemorrhage c. Complicated migraine d. Multiple sclerosis

d. Multiple sclerosis

A 40-year-old woman has recurring 30-minute episodes of disabling vertigo. The episodes are accompanied by roaring tinnitus, aural pressure, and low-frequency hearing loss. Her physical examination is normal. What is the most likely diagnosis? a. BPPV b. Acute suppurative labyrinthitis c. Acute serous labyrinthitis d. Ménière disease e. Vertebrobasilar insufficiency

d. Ménière disease

A 30-year-old woman with no past medical history presents with a productive cough of 2 weeks' duration. She states she also has had a runny nose, body aches, congestion, and fevers for the past week. In the office, she is normotensive, with a normal pulse and a temperature of 101.2 °F. Her physical examination is significant for sinus tenderness, boggy nasal turbinates, and crackles in the left lower lobe lung fields. Which of the following is the best initial step in management? a. Reassure the patient that she likely has a viral infection, and it will resolve on its own b. Order a rapid strep test and treat if positive c. Prescribe amoxicillin for a likely bacterial infection d. Order chest x-ray to rule out possible pneumonia

d. Order chest x-ray to rule out possible pneumonia

A 45-year-old man was admitted for acute pancreatitis, thought to be a result of blunt abdominal trauma. After 3 months, he still has epigastric pain but is able to eat solid food. His amylase level is elevated at 260 IU/L. Which of the following is the most likely diagnosis? a. Recurrent pancreatitis b. Diverticulitis c. Peptic ulcer disease d. Pancreatic pseudocyst

d. Pancreatic pseudocyst

A 59-year-old diabetic woman had suffered an acute anterior wall MI. Five days later, she gets into an argument with her husband and complains of chest pain. Her initial ECG shows no ischemic changes, but serum cardiac troponin I levels are drawn and return mildly elevated at this time. Which of the following is the best next step? a. Use thrombolytic therapy. b. Treat with percutaneous coronary intervention. c. Perform coronary artery bypass. d. Perform serial ECGs and obtain CK-MB.

d. Perform serial ECGs and obtain CK-MB.

A 72-year-old man with a long history of hypertension presents to the emergency department complaining of not being able to urinate for the last 36 hours. He also has a 1-day history of nausea, vomiting, and abdominal pain. He states that his urinary stream has been decreasing over the past 2 years. On examination, the abdomen is firm and tender, and the prostate is enlarged. His serum Cr level is 3.4 mg/dL. Which of the following is the best next step? a. Give intravenous fluids and see if he begins to make urine. b. Perform a renal ultrasound in the emergency department. c. Maintain tight control of his blood pressure. d. Place an indwelling Foley catheter.

d. Place an indwelling Foley catheter.

A 17-year-old adolescent male presents to the emergency department with a temperature of 101 °F (38.3 °C), a deep nonproductive cough, and generalized malaise for 3 days. He does not recall being around any particular sick contacts but is around many people in his afterschool job and at school. He states that he never had the chicken pox and is unaware of what immunizations he received as a child. He was diagnosed at age 12 with leukemia but has since been healthy. He is worried that his cancer may no longer be in remission. A chest x-ray reveals bilateral, diffuse infiltrates. Which of the following is the most likely cause of illness? a. Pneumonia caused by S. pneumonia b. Pneumonia caused by P. jiroveci c. Pneumonia caused by L. pneumophila d. Pneumonia caused by M. pneumonia e. Pneumonia caused by H. influenza

d. Pneumonia caused by M. pneumonia

A patient with which of the following conditions requires antimicrobial prophylaxis before dental surgery? a. Atrial septal defect b. Mitral valve prolapse without mitral regurgitation c. Previous coronary artery bypass graft d. Previous infective endocarditis

d. Previous infective endocarditis

A 45-year-old woman is noted to have dizziness, a pounding feeling in her chest, and fatigue of 3 hours' duration. On examination, she is noted to have a blood pressure of 110/70 mm Hg and a heart rate of 180 bpm. On ECG, she has AF, and a prior baseline ECG showed delta waves. The emergency department provider counsels the patient regarding cardioversion, but the patient declines. Which of the following is the best therapy for her condition? a. Digoxin b. Angiotensin-converting enzyme inhibitor c. Calcium channel blocker d. Procainamide

d. Procainamide

What postexposure prophylaxis (if any) should the student described in Question 26.1 receive (for hepatitis B and C)? a. HBIg b. Oral tenofovir c. Immunoglobulin d. Reassurance

d. Reassurance

An 18-year-old woman is brought to the emergency center because she fainted at a rock concert. She apparently recovered spontaneously, did not exhibit any seizure activity, and has no medical history. Her heart rate is 90 bpm, and blood pressure is 110/70 mm Hg. The neurologic examination is normal. A pregnancy test is negative, and an ECG shows normal sinus rhythm. Which of the following is the most appropriate management? a. Admit to hospital for cardiac evaluation. b. Obtain an outpatient echocardiogram. c. Use 24-hour Holter monitor. d. Reassure the patient and discharge home.

d. Reassure the patient and discharge home.

A 45-year-old man has hypertension. A thiazide diuretic agent had been prescribed with continued elevated blood pressure. The inclusion of spironolactone to the thiazide diuretic is done to achieve which of the following? a. Reduce hyperuricemia b. Reduce Mg+ loss c. Decrease the loss of Na+ d. Reduce K+ loss

d. Reduce K+ loss

A 56-year-old woman with history of hypertension, diabetes, and newly diagnosed polycystic kidney disease presents for follow-up for hypertension. Routine laboratory work shows elevated calcium of 13 mg/dL and an elevated phosphate level. The patient denies weight loss, is taking only metoprolol for her blood pressure, and denies recent history of immobilization. Given these findings, which etiology of hypercalcemia would you be most concerned about in this patient? a. Primary hyperparathyroidism, as this is the most common etiology of hypercalcemia b. Iatrogenic hypercalcemia secondary to medications c. A primary vitamin D deficiency, given her age d. Secondary hyperparathyroidism due to renal disease

d. Secondary hyperparathyroidism due to renal disease

Several friends develop vomiting and diarrhea 6 hours after eating food at a private party. They describe the diarrhea as watery and nonbloody. Which of the following is the most likely etiology of the symptoms? a. Rotavirus b. Giardia c. Escherichia coli d. Staphylococcus aureus e. Cryptosporidium

d. Staphylococcus aureus

A 42-year-old man with polycystic kidney disease who complained of a sudden onset of severe headache and then lost consciousness a. Migraine headache b. Tension headache c. Cluster headache d. Subarachnoid hemorrhage e. Meningitis

d. Subarachnoid hemorrhage

68-year-old male with hypertension presents for annual examination. On review of systems he reports urinary hesitancy and nocturia. Your examination reveals a nontender but enlarged prostate without nodules. On review of his blood pressure logs and clinic readings he is averaging values of 150/80 mm Hg. Which of the following medications would offer treatment of the hypertension and prostatic symptoms? a. Furosemide b. Aliskiren c. Propranolol d. Terazosin

d. Terazosin

An 18-year-old adolescent young man is brought to the emergency department from his college dorm by his roommate. The patient is confused and cannot give a history. He has a temperature of 104 °F (40 °C), pulse of 110 beats/min, blood pressure of 90/60 mm Hg, and respiratory rate of 24 breaths/min. His head cannot be moved because of severe nuchal rigidity. Multiple petechiae are observed on his buttocks and legs. After initiation of treatment for the patient, what is the most appropriate advice to give to this patient's roommate? a. Provide reassurance that he does not require prophylaxis. b. The patient should take acyclovir for prophylaxis. c. The patient should take penicillin for prophylaxis. d. The patient should take rifampin for prophylaxis. e. The patient should take cefuroxime for prophylaxis.

d. The patient should take rifampin for prophylaxis.

A 45-year-old African American woman presents for a routine examination. You notice a 9-mm diameter lesion on the palm of her right hand that is dark black and slightly raised and has a notched border. When asked about it, she says that it has been present for about a year and is growing. A friend told her not to be concerned because, "Black people don't get skin cancer." Which of the following is your advice? a. Her friend is correct, and this is nothing to worry about. b. While anyone can get skin cancer, this lesion has primarily benign features and can be safely observed. c. This lesion is suspicious for cancer, but this is most likely a metastasis from breast cancer. d. This lesion is suspicious for a primary melanoma and needs further evaluation immediately.

d. This lesion is suspicious for a primary melanoma and needs further evaluation immediately.

A 48-year-old man is being seen for follow-up from an elevated cholesterol level at a health screening, which is recorded as 250 mg/dL. He denies significant medical history and has no symptoms. Which of the following tests is the most important further evaluation of this patient's condition? a. Electrocardiogram (ECG) b. Stress test c. Complete blood count (CBC) d. Thyroid-stimulating hormone level

d. Thyroid-stimulating hormone level

A 55-year-old man is noted by his family members to be forgetful and become disoriented. He has difficulty making it to the bathroom in time and complains of feeling as though "he is walking like he was drunk." Which of the following therapies is most likely to improve his condition? a. Intravenous penicillin for 21 days b. Rivastigmine c. Treatment with fluoxetine for 9 to 12 months d. Ventriculoperitoneal shunt e. Enrollment into Alcoholics Anonymous

d. Ventriculoperitoneal shunt

A 35-year-old morbidly obese woman patient is being evaluated in the emergency center from an outlying clinic. She was seen 8 days ago for headache, fever of 102 °F, nonproductive cough, and myalgias, for which she was diagnosed with influenza and prescribed oseltamavir for 5 days. She felt better after taking the medication initially but now feels she is getting worse. She is sent to the emergency department for expedited evaluation. She states that she has had night sweats, chills, shortness of breath, and cough productive of yellowish-green sputum for 3 days. Today, her vital signs show a temperature of 104 °F, with a respiratory rate of 30 breaths/min, heart rate of 100 bpm, and pulse oximetry of 93% on room air. Assuming admission for pneumonia, which of the following is the best empiric antibiotic treatment for this patient? a. Continuation of oseltamivir for another week b. Azithromycin c. Penicillin d. Levofloxacin e. Ceftriaxone with vancomycin

e. Ceftriaxone with vancomycin

A 24-year-old man who has sex with men presents for his yearly health maintenance examination. He wants to be "tested for everything" but is completely asymptomatic. His BMI is 22 kg/m2, blood pressure is 125/76 mm Hg, and he has no significant medical history. According to the USPSTF, for which of the following should this man be screened? a. Testicular cancer b. Marijuana abuse c. Diabetes d. Hyperlipidemia e. Depression

e. Depression

A 7-year-old boy is brought to a hospital in Charlotte, North Carolina, with a fever of 104 °F (40 °C). A maculopapular rash is seen on his wrists and ankles, but the palms and soles are spared. His laboratory results show leukopenia, hyponatremia, and elevated liver transaminases. His parents say that he was on a camping trip 1 week ago, but they vigorously used insect repellants and filtered all of their water. His father came in contact with poison oak, but the boy denies any pruritus. Which of the following is the best treatment for this patient's rash? a. Penicillin b. Acyclovir c. Ceftriaxone d. Vancomycin e. Doxycycline

e. Doxycycline

A 13-year-old boy has a nonproductive cough and mild shortness of breath on a daily basis. He is awakened by the cough at least 5 nights per month. Which one of the following would be the most appropriate treatment for this patient? a. A LABA daily b. A SABA daily c. Oral prednisone daily d. An oral leukotriene inhibitor as needed e. ICS daily

e. ICS daily

A 65-year-old dialysis patient is found to have a serum potassium level of 6.8 mEq/L, which is verified on a stat repeat level. An ECG shows peaked T waves and a widened QRS complex. What is the first intervention that should be made at this point? a. Intravenous glucose and insulin administration b. Arrangement for a dialysis treatment c. Oral sodium polystyrene (Kayexalate) d. Intravenous furosemide e. Intravenous calcium

e. Intravenous calcium

A 34-year-old man is being seen in the office for follow-up for epigastric pain and suspected PUD. Serum antibodies are positive for H. pylori. Which of the following is the most accurate statement regarding this infection? a. It is more common in North America than in the developing world. b. It is associated with the development of colon cancer. c. Eradication of H. pylori eliminates most cases of nonulcer dyspepsia. d. It is believed to be sexually transmitted. e. It is a cause of both duodenal and gastric ulcers.

e. It is a cause of both duodenal and gastric ulcers.

A 22-year-old college student with fever, headache, photophobia, and CSF with 25 white blood cells per high-power field, but no red blood cells or xanthochromia a. Migraine headache b. Tension headache c. Cluster headache d. Subarachnoid hemorrhage e. Meningitis

e. Meningitis

A 45-year-old man with no significant past medical history presents with severe back pain after lifting heavy boxes at work 2 days ago. Other than his back pain, his review of symptoms is negative. His pain is exacerbated by coughing and sneezing. The pain radiates from his lower back down his right posterior thigh to his great toe when you perform both a straight leg raise and the contralateral leg raise tests. His strength, sensation, and reflexes are intact and symmetrical. Which of the following is the recommended first step in the evaluation of this patient? a. Plain film radiographs b. MRI c. CT scan d. Nuclear bone scan e. No imaging indicated

e. No imaging indicated

A 34-year-old man with a 20-year past history of asthma presents to an acute care clinic with an asthmatic exacerbation. Treatment with nebulized albuterol and ipratropium does not offer significant improvement, and he is then admitted to the hospital. He is afebrile and has a respiratory rate of 24 breaths/min, pulse rate of 96 beats/min, and oxygen saturation of 93% on room air. On examination, he has diffuse bilateral inspiratory and expiratory wheezes, mild intercostal retractions, and a clear productive cough. Which one of the following is the best next step in the management of this patient? a. Chest physical therapy b. ICSs c. Azithromycin orally d. Theophylline orally e. Oral corticosteroids

e. Oral corticosteroids

A 64-year-old woman with a history of hypertension and angina pectoris presents with chest pain for the last 3 hours. She describes the pain as "sharp"; it is worse when she inhales deeply, and it is not relieved by sublingual nitroglycerin. The pain improves if she sits up and leans forward. Her ECG shows ST elevation in most leads. Cardiac enzymes are negative. Which of the following is the most likely diagnosis in this patient? a. Unstable angina pectoris b. Myocardial infarction c. Aortic dissection d. Heart failure e. Pericarditis

e. Pericarditis

Which of the following therapies is most likely to provide the greatest benefit to a patient with chronic stable emphysema and a resting oxygen saturation of 86%? a. Inhaled tiotropium daily b. Inhaled albuterol as needed c. Oral prednisone daily d. Supplemental oxygen used at night e. Supplemental oxygen used continuously

e. Supplemental oxygen used continuously

A 7-month-old infant is brought by her mother to an outpatient clinic because of a 2-day history of fever, copious nasal secretions, and wheezing. The mother volunteers that the baby has been healthy and has not had these symptoms in the past. The infant's temperature is noted to be 100.7 °F (38.1 °C), her respiratory rate is 50 breaths/min, and her pulse oximetry is 95% on room air. Physical examination reveals no signs of dehydration, but wheezing is heard on bilateral lung fields on auscultation. The infant shows no improvement after three treatments with nebulized albuterol. Which of the following is the recommended treatment? a. Continued nebulized albuterol every 4 hours b. Antihistamines and decongestants c. Antibiotics for 7 days d. Initiate palivizumab e. Supportive care with hydration and humidified oxygen

e. Supportive care with hydration and humidified oxygen

A 5-year-old girl developed high fever, ear pain, and vomiting a week ago. She was diagnosed with OM and started on amoxicillin-clavulanate. On the third day of this medication she continued with findings of OM, fever, and pain. She received ceftriaxone intramuscularly and switched to oral cefuroxime. Now, 48 hours later, she has fever, pain, and no improvement in her OM; otherwise she is doing well. Which of the following is the most logical next step in her management? a. Addition of intranasal topical steroids to the oral cefuroxime b. Adenoidectomy c. High-dose oral amoxicillin d. Oral trimethoprim-sulfamethoxazole e. Tympanocentesis and culture of middle ear fluid

e. Tympanocentesis and culture of middle ear fluid

A 54-year-old woman is being evaluated in the emergency center with shortness of breath of 12 hours' duration. She also has significant vaginal bleeding of 1 month's duration. On examination, she is found to have significant pallor of her sclera and skin. Speculum examination showed a large necrotic and exophytic mass of the cervix. The hemoglobin level is 7 g/dL. Her left leg is swollen and markedly different from her right leg. Doppler investigation reveals a DVT of the left leg. Which of the following is the best treatment for the thrombus? a. Intravenous unfractionated heparin b. Fractionated subcutaneous heparin c. Subcutaneous unfractionated heparin d. Oral warfarin (Coumadin) e. Vena cava filter

e. Vena cava filter

A 18-year-old woman eats raw seafood and 2 days later develops fever, abdominal cramping, and watery diarrhea. She is being seen in the emergency center for dizziness and lightheadedness. She describes the diarrhea as profuse and continues even if she is not eating or drinking foods. Which of the following is the most likely etiology of the symptoms? a. Rotavirus b. Giardia c. Escherichia coli d. Shigella species e. Vibrio species

e. Vibrio species


Ensembles d'études connexes

FIN 323 Chapter 6 Homework Practice for Exam

View Set

Lesson 2.4 - Elementary Matrices

View Set

Writing, Speaking, and Listening Part 3

View Set

Economics-Unit 3.3: The role of trade unions

View Set

THE SOLAR SYSTEM : all homework questions

View Set

Healthcare Accounting Final Study Guide

View Set

intro to Business entrepreneur final exam review

View Set